SlideShare a Scribd company logo
1 of 107
INTERNATIONAL 
FINANCIAL 
MANAGEMENT 
Prof. Parveen Sultana Kanth
RETURN ON INVESTMENT - IFM
PROBLEM 
 An American investor purchased securities in Indian 
market investing one million dollars, at the time of 
investing exchange rate is 50/50.5 Rs while one 
year latter the exchange rate is 53/54 Rs . what is 
the rate of return to American investor if he rate of 
Rs return on Indian security is 20% 25% & 50%.
 If the rate is = 20% 
 Amount invested in India = 
10,00,000 * 50 = 50,000,000 (50 million) 
Return = 100,00,000 (1 million) 
After 1 year the amount to be repatriated = 
60000000/54= 11,11,111. 
 Rate of return= 11,11,111/10,00,000 = 11.11% 
if the rate = 25% 
 Amount invested is 10,00,000 * 50 = 5,00,00, 000 
at 25% return rate of return in Indian rupees = 5,00,00,000 
*25% = 1,25,00,000 
at the closing rate = 6,25,00,000/54 = 11,57,407 
Return = 157407/10,00,000 = 15.47%
 If the rate is 50% in Indian Rs term 
 Investment = 5,00,00,000 
 Return in Indian Rs = 5,00,00,000*50 = 2,50,00,000 Rs 
 Total amount to be repatriated in Rs = 7,50,00,000/54 
= 13,88,889$ 
 Return on investment = 3,88,889/10,00,000 = 38.89%
PROBLEM :-2 
 Risk free return in India is 8%, an American investor 
wants to invest in Indian securities with beta of 2 
with variance of 15%while exchange rate of rupees 
deprecation by 10% with variance of 20%. Market 
related return is 18% and correlation coefficient 
between return on security and exchange rate is 
25% ,find out the expected rate of return and risk in 
investment
CAPM THEORY IS APPLIED 
 As per CAPM the expected return on securities = 
risk free interest rate +beta (market rate –risk free 
interest rate). 
= .08 + 2 (.18 - .08) = 28% 
If 100 $ are invested in market then return is 128 $, 
then excluding exchange rate. 
variance of return = .15+.20+2* .25s square root of 
(.15* .20) 
= .0.44 = 44% of risk
FOREIGN EXCHANGE MARKET
PROBLEM:-1 CROSS RATE 
 The Germany DM is selling for $ 0.62 & the buying 
rate for the French franc (FF) o.17$, what is the 
FF/DM cross rate ? 
 US$0.62 FF FF 3.65 
------------- x -------- = --------------- 
DM US$ 0.17 DM
PROBLEM:-2 CROSS RATE 
 The US$ Thai Bhat exchange rate is US$ 
0.02339/Bhat, and the US$ Indian Rupees is 
US$0.02538/INR . Suppose that INR is not quoted 
against Thai Bhat . What is the Bhat/INR? 
US$0.02538 US$ 0.02339 US$ 0.02538 Bhat 
---------------- + --------------- = ------------- x --------- 
INR Baht US$ 0.02339 INR 
Ans : Baht 1.085 
---------------- 
INR
THE SPOT PRICE = 0.025063,USD/INR , AND FORWARD 
RATE IS 0.02439, USD/INR FOR 6 MONTHS FORWARD , THE 
ANNUALIZED PREMIUM IS AS FOLLOWS SELLING AT PREMIUM 
 Forward premium spot rate- forward rate 360 
Or discount = ---------------------------- x ------- 
forward rate days 
 For 6 months forward , the annualized premium is as 
follows 
0.025063 - 0.02439 360 
= -------------------------- x ------ = 5.5% 
0.02439 180
 Forward rupees is selling at a discount of 5.5%relative to 
the dollar for 6 months. In terms of indirect quote , the 
INR/$ spot exchange rate is INR 39.90/$ and forward rate 
is INR 41.00/$. For indirect quote, the forward premium or 
discount can be calculated as follows: 
 Forward premium Forward rate - spot rate 360 
or discount = --------------------------- x ----- 
spot rate days 
41.00 -39.90 360 
= ---------------- x ------ 
39.90 180 = 5.5%
Balance of payment, Cross 
Rate , Spot Rate Forward 
Rate, exchange rate theories
PROBLEM:-3 
An exporter has to surrender $ 100,000 to bank . Rate of dollar 
against INR is 46/46.15, the bank charge a commission of 
.15%. If the transit time fixed by RBI is 20 day and the rate of 
interest charge by the bank is 10%. 
 Find out the net proceeds to be credited to exporters 
account? 
 Find out what is the amount to be paid by the person if 
he is an importer instead of exporter? 
 If Rs $ rate is 46/46.15 $/pound rate is 1.71/1.74 what is 
Rs pound rate. 
 If French franc $is 10.17/10.20 and Swiss franc $ rate is 
1.73/1.75. find out French franc / Swiss franc rate? 
 If one Rs can buy 3.23 yen and if one Rs can buy .028 $ 
how many yens can 1$ buy? 
 In London a dealer quotes Dm/pound spot = 
3.2550/3.2555,Yen pound spot = 180/181 . What do you 
expect yen/Dm rate in Frankfort?
 Find out the net proceeds to be credited to 
exporters account? 
Solution: since the exporter sells the dollar to bank 
bid price of 46 per $ to be taken . since it involves 
buying of $ by bank commission should be 
deducted from exchange rate. 
Rate to be adopted is = 46 - (46 * .15%) =45.93 
Total amount is 100,000* 45.93 = 45,93,000 
Interest charged is 10% for 20 days on the net 
proceeds. = 45,93,000 * 10 *20/365 = 25,167 Rs 
Net proceeds to be credited to the account of 
exporter = 45,93,000 - 25,167 = 45,67,833
Find out what is the amount to be paid by the person if he 
is an importer instead of exporter? 
 Solution: Since in this case the bank is selling the dollar 
to importer , it charges the ask rate of 46.15 and the 
commission is added instead of being deducted . 
Amount to be paid by the importer 
= 46.15+(Rs 46.15*.15%) = 46.22 Rs 
The amount to be paid by the importer to bank = $ 
1,00,000* 46.22 = Rs 46,22,000.
 If Rs $ rate is 46/46.15 $/pound rate is 1.71/1.74 what is 
Rs pound rate. 
Rs/pound rate = Rs/$* $/pound (cross rate) 
= 46*1.71/46.15*1.74 
= 78.66/80.30 
 If French franc $is 10.17/10.20 and Swiss franc $ rate is 
1.73/1.75. find out French franc / Swiss franc rate? 
 French franc /Swiss franc rate = French franc /$ * Swiss 
franc/$. 
Swiss franc /$c rate = 1/1.75/1/1.73 = .5714/.5780 
French franc /Swiss franc = 10.17*.5714/10.20*.5780 
= 5.81/5.90
 If one Rs can buy 3.23 yen and if one Rs can 
buy .028 $ how many yens can 1$ buy? 
= .028 $ = 3.23 yen 
1 $ = ? 
$ = 3.23/.028 = 115.35 yen
 In London a dealer quotes Dm/pound spot = 
3.2550/3.2555,Yen pound spot = 180/181 . What do you 
expect yen/Dm rate in Frankfort? 
Sol: Yen/Dm spot rate = yen /pound spot rate * pound /Dm 
spot rate. 
Dm/pound spot rate = 3.2550/3.2555 
Pound /Dm spot rate = 1/3.2550/1/3.2555 = .3072/.3071 
Yen/Dm spot rate = 181* .3072/181*.3071 
= 55.603/55.278 
 Frankfort get quote of yen/Dm spot 51.1530/51.2550 is 
there an arbitrage opportunity 
Sol: Buy 1Dm in Frankfort by selling 51.2550 yen, sell 1 
Dm and get 55.7278 yen there by making profit of 
55.278 - 51.2550 = 4.023 yen
 January , 24, 2014 a customer requested a bank to 
remit DG(Decigram) 250,000 to Holland as 
payment for Diamonds, However due to strike the 
bank can make payment only on Feb ,1,2014. 
Inter bank rates are as follows: 
Jan 24,2014 Feb 1 2014 
Mumbai $/Rs (100Rs) 
3.10/3.15 
3.07/3.12 
London $/pound 
1.7250/60 
DG /pound 
3.957/90 
1.7175/85 
3.9380/90 
How much does the customer stand 
to gain or lose due to delay? 
PROBLEM
 Sol: Rs/Dg = Rs/$* $/Dg 
Rs/$ = 100/3.15/100/3.10= 31.75/32.26 
= 100/3.12/100/3.07= 32.05/32.57 
$/Dg =$/pound*pound/Dg 
= 1.7250/3.9590/1.7260/3.9575 = .4357/.4361 
= 1.7175/3.9390/1.7185/3.938 = .4360/.4364 
Rs /Dg = 31.75 * .4357/32.26 *.4361 = 13.83/14.07 
= 32.05 *.4360/ 32.57 * .4364 = 13.97/14.21 
The importer has lost (14.21 – 14.07) * 250,000 Dg = 
Rs 35000 due to delay remittance
PROBLEM 
 Consider the value of DEM relative to USD. The 
spot rate is DEM 1.82. the interest rates in the US 
and Germany are 5% and 3% respectively. 
Estimate the price on 4 month forward contract on 
DEM. 
 Solution: 
Spot rate of Dem (1USD= DEM) 1.82 
Interest rate US =5% 
Interest rate in Germany= 3%
SOLUTION 
 Et =Eo (1+rhc/1+rfc)t. 
Where et = expected forward rate for time t. 
Eo = spot rate 
rhc =rate of interest in home currency. 
Rfc =rate of interest in foreign currency. 
T= time period. 
1.82( 1+ 0.03/1+ 0.05) x 4//12 = 0.595
PROBLEM 
USD/INR spot =48.75/80, 2 months swap =.12/.20. 
USD?JPY spot=125.50/126.10, 2 month swap =.20/.15. 
Find INR/JPY 2 month outright ? 
Solution: USD/INR spot = 48.75/48.90 
2 month = 48.87/ 49.90. 
USD/JPY spot = 125.50/ 126.10 
2 month = 125.30/ 125.95 
INR & JPY = 125.30/48.87 = 2.56. 125.97/ 49 = 2.57. 
JPY = 2.56/2.57 for 1 INR
PROBLEM 
Give the following date, calculate the 
arbitrage possibilities 
Spot rate : 42.0010=1$ 
Forward rate (6month)= 42.8020Rs/1$ 
Annualised interest rate on 6 month Rs:12% 
Annualised interest rate on 6 month dollar:8%
SOLUTION 
 The rule is that if the interest rate differential is greater 
than the premium or discount ,place the money in the 
currency that has a higher rate of interest or vice versa 
 Negative interest rate differential = 12-8=4% 
 Forward premium = forward rate –spot rate 
------------------------------- X 100 
spot rate 
42.8020-42.001 12 
---------------------- x100 x -------- = 3.8141% 
42.0010 6
 Negative differential > forward premium . Hence, there is 
a possibility of arbitrage inflow in India. 
 Arbitrage possibility for an investment of $1000 by taking 
a loan @ 8% in US. An arbitrageur would invest in India 
at spot rate of RS 42.0010 @12% for six months and 
cover the principal +interest in six month forward rate 
 Principal:$1000= 42,001 Rs 
 Interest on investment for six month = Rs 42001 x 
12/100 x 6/12 = 2520.06 
So total amount at the end of six month+ principal +interest 
= 2520.06+ 42001= 44,521.06
 Converting the above in dollars at the forward rate = 
44,521.06/42.8020= $1040.16 
 The arbitrageur will have to pay at the end of six month = 
$ 1000 + ($1000 x 8/100 x6/100) = $1000+ $40 =1040$ 
 Hench the arbitrageur gain + ($1040.16-$1040) =$0.16 
on borrowing $1000 for six month
PROBLEM: INTEREST RATE PARITY 
An American firm purchases $4000 worth of perfume (ff 20,000) 
from a French francs. The following quotation and 
expectations exist for the FF. 
 Present spot rate $ 0.2000, US interest rate15%. 
 90 day forward rate 0.2200, French interest rate 10% 
SR 90days = 0.2400. 
 What is the premium or discount on the forward French francs 
? What is the interest differential between US and French ? Is 
there an incentive for covered interest arbitrage? 
 If there is CIA (covered interest arbitrage), how can an 
arbitrageur take advantage of the situation? Assume the 
arbitrageur is willing to borrow $4000 or FF 20,000 and there 
are no transaction costs. 
 If transaction costs are $50 , would an opportunity still exit for 
CIA?
SOLUTION 
 Forward premium on FF = FR-SR/SR X 100 
.22-.20 
------------ X 100 X 4 = 40% 
.20 
Interest rate different is 15-10=5% . 
Yes there is an incentive for CIA (outflow of trends 
from US ) as interest differential in favour of France 
is -5% in flour of US 5%.
 The arbitrageur can take advantage of the situation in the 
following manner. 
Borrow $4000 for 90 days. So amount is t paid after 90 
days 
= 4000 (1+15% X1/4) =$4.150 
Convert $4000into FF at current SR ie $1=5Ff 
$4000 = FF 4000x5 =FF 20,000 
$4000= FF 20,000 
Invest 20,000 in France @10% p.a for 90 days 
Amount received at the end of 90 days = FF 20,500
 Sell investment proceed forward at rate FF1= $.22 
Amount received in US $ after 90 days by selling FF 
20500= $ (20500 x .22) = $4510 
Amount received =4510$ 
Amount paid =4150$ 
------------ 
Profit = 360$ 
 As the profit of 360$ > transaction cost of 50 $ 
opportunity still exist for a CIA.
PROBLEM 
Dm spot was quoted 0.40$ in New York, the 
price of the pound sterling was quoted 
$1.80. 
1. what would you expect the price of the 
pound to be in Germany? 
2. If the pound were quoted in Frankfort at 
Dm 4.40/pound what would you do to 
profit from the situation?
SOLUTION 
 1Dm =$ 0.40 ; 1 pound = $1.80 
1/0.40Dm = 1/1.80pound 
1 pound = 1.80/0.4 Dm = 4.5Dm 
= 4.5 Dm/pound. 
 Buy 1 pound for Dm 4.40 in Frankfort . With this 
pound sell in NY: for $1.80. with $1.80 , buy 4.5 
Dm . Hench the arbitrager will profit 0.1 Dm for 
every 4.40 Dm
PROBLEM 
 You have called your foreign exchange market and asked for 
quotations on the spot , 1 month, 3 months, 6 months. The 
trader has responded with the following. 
= $ 0.024.479/81 3/5 8/7 13/10 
1. What does this mean in terms of dollars per euro 
2. If you wished to buy spot Euros how much would you pay in 
dollars? 
3. If you wanted to purchase spot US$ how much would you 
pay in euro? 
4. What is the premium or discount in the 1, 3, 6 months 
forward rates in annual percentages? ( assume you are 
buying in Belgian francs) 
5.
SOLUTION 
Spot rate Bid price Ask price 
Spot rate $0.2479 $0.2481 
1 month $0.2482 $0.2486 
3 months $0.2471 $0.2474 
6 months $0.2466 $0.2471 
2. $ 0.2481 Will Ask Price 
3. US $ spot = 1/ 0.2481 = 4.034
 Premium or discount rates in forward market (assuming that buying 
in Belgian francs) 
0.2486 – 0.2481 
1 month = ------------------------ X 12 X100 = 2.42% 
forward 0.2481 
premium 
0.2474 – 0.2481 12 
3 month = ------------------------ X 100 X ----- =1.13% 
Forward 0.2481 3 
Discount 
6 month = 0.2471- 0.2481 12 
Forward --------------------- X 100 X ----- = 0.8% 
Discount 0.2481 6
PROBLEM 
 Spot rate = 44.0030 = $1 
 6 month FR = 45.0010 = $1 
 Annualised interest rate on 6 month rupee = 12% 
 Annualised interest rate on 6 month dollar = 8% 
Given the above data is there arbitrage possibility?
SOLUTION 
 6 month forward US $ is being quoted at a 
premium as follows 
45.0010- 44.0030 12 
------------------------- X ------ X100 = 4.5361% 
44.0030 6 
Interest rate differential = 12-8 = 4% 
Since interest rate differential is smaller than the 
premium , it would be beneficial to place money in 
US $ as 6 month interest rate is lower .
An arbitrageur would take the following steps: 
1. Borrow Rs10,000 at 12% for 6 months 
2. Convert this SR to obtain US$ 227.257 (10,000/44.0030) 
3. Invest $ at 8% in money market for 6 months to receive 
6 1 
= $ 227.257 8 X ----- X ----- + 1 
12 100
4. Sell Us$ at 6 month forward to receive 
= 236.3473 X 45.0010 = Rs 10635.865 
5. Return the rupee debt borrowed at12 % . The 
amount to be refunded is Rs 10,600 
6 1 
=10,000 1+ 12 X ---- X ----- + 1 
12 100 
Profit = amount received – amount borrowed 
= 10,635.865 -10,600 = 35.865
PROBLEM 
 From the following data calculate the possibilities of 
gain /loss in arbitrage. 
Spot rate FFr 6.00 = $1 , 6 month forward rate FFr 
6.0020 = $1. 
Annualised interest rate on 6 month s US$ = 5% 
Annualised interest rate on 6 month Fr = 8% 
(It Direct Quote Method)
SOLUTION 
 Negative interest rate differential = 5 - 8 = -3% 
forward Premia (annualized ) = FR-SR/SR *100 * 12/6 
= (6.0020 -6.0000 /6.0000) x100 x 12/6 = 0.67% 
Here we find that the negative interest rate differential > 
forward Premia , Hench there will be arbitrage inflow in 
France
 The arbitrage possibility will be shown below. 
1. Assume an arbitrageur borrows $1000 for 6 months. 
Amount to be repaid at the end of 6 months will be = 
$1000 +$ 1000 x5/1000 x6/12 = $1025. 
2. The arbitrage would then convert $ into FF at spot rate 
and invest the amount in France @8%. Converting $ 
into FF at the spot rate $100 –FF 6000. 
Invest in France @8% = 6000 x 8/100 x 6/12 = 240 
Total amount received is FF 6000 + FF240 = FF 6240. 
convert Ff6240 into $ at the forward rate at the end of 6 
month = FF 6240 /FF 6.0020 = $ 1039.65. 
Amount received by the arbitrageur = $1039.65 
Amount paid = $ 1025.00 
----------------------- 
profit = $ 14.65
Problem: 
 If the current Rs/$ rate is Rs 50, one year inflation rate 
is 10% for Rs and 2% for dollar. If 1 year later , the rate 
of exchange between $ & Rs in forward rate is 55, is 
there scope for arbitrage , if PPP is applicable? 
 Spot exchange rate between Rs/$ 50, if annual inflation 
rate is 10% in India and 3% in us what will be the rate of 
return to Indian investor if the return on us security is 
10%if PPP is applicable? 
 In the above case if the percentage of return in India is 
10%. What is the rate of return to American investor?
Sol :1: S1A/B = S0A/B X (1+rA/1+rB) 
S1 = Future Exchange Rate 
rA = Interest rate of country A 
rB = Interest rate of country B 
As per PPP one year later rate will be = 
(50* 1.10)/1.02 = 53.92. 
Since the present forward rate is not in alignment he can 
sell the $ forward 
 Sol: 2: Indian investor can convertRs50 to one dollar 
and invest the same in Us security. After one year he will 
get 1.10 $ . 
 One year later the= $=(50* 1.10)/1.03 =53.40 Rs. 
 Sol:3: American investor can convert one dollar to 50Rs 
and invest the same in Indian market. After one year he 
will get Rs 50 * 1.10 = Rs55, if this is converted to dollar 
it will be 1.03$ . So the rate will be 3%for American 
investor.
PROBLEM 
An Indian company ,AB ltd imports machinery worth 
₤2.0 million and is to make the payment after 6 
months. The current rate are 
Spot rate = Rs 66.96/₤ 
6 month forward rate = Rs 67.50 /₤ 
 What should AB ltd do if they expect that in six 
month time the pound will settle at Rs67.15/₤? 
 What are the option available to the company in 
case of an expected appreciation / depreciation in 
the rupees?
SOLUTION 
 Spot rate ₤1 = Rs 66.96 
 6 month forward rate ₤1 = 67.50 
 Expected spot rate after 6 month ₤1 = Rs 67.15 
1) Since AB ltd has a liability in foreign currency pound , 
they are importing a machinery worth ₤ 2.0 million . Both 
the market and the company expect the pound to 
appreciate. 
2) Hench company should estimate the relative cost of 
hedging and if it is not high , the company should hedge 
its payments. 
3) Incase of depreciation of pound the company need not 
do anything as it stands to gain . In case of appreciation 
of pound it should hedge its payments as the company 
will be exposed to exchange rate risk
PROBLEM 
 The following data is given: 
Spot rate FFrl = 66.60 Rs 
6 month forward rate FFrl = Rs 6.85 
FFr interest = 8.3% 
Rs interest = 10.5 % 
Analyze the different arbitrage possibilities
SOLUTION 
 Given the above data if one invest money in India , he 
gets = ( 1+ .105/2) =1.0525 after 6 months. 
 If one invest money in FFr , he gets 6.85/6.60 
(1+0.83/2)= Rs10809 
 So there will be an arbitrage outflow from india.
PROBLEM 
 Set out below is a table of cross rates. 
Deutsch 
mark 
Dollar French 
franc 
Pound 
sterling 
Frankfurt ? 2.2819 0.4712 4.0218 
New York 0.4421 ? 0.2110 1.8000 
Paris 2.0949 4.7393 ? 8.4301 
London 4.0207 1.7775 8.4232 ? 
For Frankfurt , New York and Paris all quotes for London all quotes 
are indirect. If all above quotes were available at the same time and 
assuming no transaction costs how might a trader take advantage of 
the situation
SOLUTION 
Given the arbitrage opportunities with no transaction cost. 
 Arbitrage between Dm & pound : buy pound in Frankfort . 
For every Dm, one will get ₤4.0218. These can be sold in 
London to get Dm for which one will have to pay ₤ 4.0207. 
These can be sold in London to get Dm for which one will 
have to pay ₤ 4.0207.Thus there is a potential gain of 
0.0011 pounds for every Dm. 
 Arbitrage between $ & ₤ : buy ₤ in New York and sell in 
London. 
 Arbitrage between FFr and ₤ : buy Paris and sell in 
London.
 Arbitrage between $ & ₤ :buy $ in Frankfort and sell in 
New York . For every Dm one will get $2.2819. these can 
be sold in Frankfort and sell in New York . For every Dm 
one will get $2.2812. these can be sold in New York to get 
Dm for which one will have to pay ₤2.2619. thus there is a 
potential gain of $0.0193 for every Dm traded. 
 Arbitrage between Ff and Dm : buy in Paris and sell in 
Frankfort. For every FF one will get Dm 2.0949. these can 
be sold in Frankfort to get Ff for which one will have to pay 
Ff 0.9817. thus there is a potential gain of Ff 0.0128 for 
every Dm traded. 
Similar arbitrage opportunities will exist across all the 
currency combinations.
PROBLEM 
 A foreign exchange trader gives the following quotes for 
the Belgian franc Spot , 1 month, 3month , 6 month to us 
based treasurer. 
= $0.02368/70 4/5 8/7 14/12. 
1. Calculate the outright quotes for 1, 3, &6 months 
forward?. 
2. If the treasurer wished to buy Belgium franc 3 month s 
forward how much would the pay in dollars ? 
3. If he wished to purchase US$ 1 month forward how 
much would he have to pay in Belgium franc? 
4. Assuming that Belgian franc are being bought what is the 
premium/ discount for the 1, 3, 6 months forward rate in 
annual percentage terms? 
5. What do the above quotation imply in respect of the term 
structure of interest rates in the USA & Belgian?
SOLUTION 
Bid Ask 
Spot $ 0.02368 $0.02370 
1 month $0.02372 $0.02375 
3 month $ 0.02360 $0.02363 
6 months $0.02354 $ 0.02358
2. To buy Belgian francs 3 month forward the treasurer has 
to pay $ 0.02363. 
3.US $ 1 month forward = 1/0.02375= Belgian franc 
42.10526. 
For 1 month forward premium 
= (0.02372-0.02368)/0.02368 x 100 x 12=2..27% p.a. 
For 3 month forward discount 
= (0.02360- 0.02368)/0.02368 X100 X12/3 = 1.35% p.a. 
For 6 month forward discount 
= (0.02354- 0.02368)/0.02368 x100 x 12/6 = 1.185 p.a. 
4. Belgian interest rates are expected to rise and US 
interest rate are expected to fall.
PROBLEM 
 “Compaque” company has to make a US $ 1 million 
payment in 3 months time . The dollars are available 
now . You decide to invest them for 3 months. 
US$ deposit rate :9% p.a. 
UK deposit rate :10% p.a. 
Present spot rate is $ 1.90/pound. 
3 month forward rate is $ 1.88/₤. 
 Where should the company invest for better returns? 
 Assuming that the interest rates and the returns spot 
exchange rate remain as above what forward rate would 
yield an equilibrium situation? 
 If the sterling deposit rate was 12% p.a. and all other 
rates remain as in the original question, where should 
you invest?
SOLUTION 
 Alternative :1 
1. invest US$ 1 million in Us @9% p.a. for 3 months. Interest earned = ( 
100,000 x.09 x 3/12) - $ 22,500. 
 Alternative :2 
1. sell the US $ 1 mn and buy pound from the spot market , we get = 
1,000,000/1.90 
= ₤ 526,315.789. 
1. Invest the available ₤ @10% for 3 months yield an interest = 
526,315.789 x 10 /100 x 3/12 = 13,157.895. 
2. principal + interest after 3 month = 526,315.789 + 13,157.895 = ₤ 
539,473.684. 
3. selling the pound to buy US $ we get 539,473.684 x 1.88 = $ 
1,014,210.526 
4. Hench income = $1,014, 210.526- $1,000,000 = $14210.526. Hench 
the company should invest in the US as it results in better return of 
US$ 8,290.
 Assume the forward rate be ‘x’ 
For an equilibrium situation amount at the end of 3 month s 
should be equal . Hench amount invested in sterling covered 
by forward rate should be = $ 1,022,500. 
Hench x = 1022,500/539473.684 = 1.895 
Hench forward rate = $ 1.89 / ₤. 
 Sell the US$ and buy ₤ from the spot market we get = 
1,000,000/1.90 = ₤ 526,315.789 
1. invest the available ₤ @12% for 3 months yield an interest = 
526,315.789 x 12 /100 x 3/12 = 15,789.473 
2. Principal = interest after 3month = 526,315.789 = 15,789.473 
= ₤ 542,105.262. 
3. Selling the pound to buy US$ we get = 542,105 .262 X1.88 = 
$ 1,019,157.893 
Thus income (net) = 19,157.893. 
1. since interest earned by investing in US$ = 22,500. Hench 
even if the sterling deposit rate become 12% p.a. investing 
US$ is still a more profitable alternative.
PROBLEM 21 
 A MNC gives the following outright quotations for 
the Singaporean dollars: 
BID ASk 
Spot rates 1.2440 1.2450 
1 month 1.2455 2.2475 
3month 1.2477 1.2484 
6 month 1.2482 1.2498
 Calculate forward quotes for the Singaporean dollar 
as an annual percentage premium or discount if 
you reside in US. 
 calculate the annual percentage premium or 
discount on the US dollar for each forward rate for a 
foreign exchange trader residing in singapore?
SOLUTION 
 We are buying USD. 
BID ASK Premium on 
buying USD 
annualized 
Spot rate 1.244 1.245 
1 month 1.2455 1.2475 0.200803214 2.409638554 
3 month 1.2477 1.2484 0.072144289 0.288577154 
6 month 1.24 82 1.2498 0.112143544 0.224287087
Bid Ask % discount 
on buying 
USD 
annualized 
Spot rate 0.80385852 0.803213 
1 month 0.80289041 0.801603 -.0.002004008 -0.0240280 
3 month 0.80147471 0.801025 - 0.000720923 -0.0028836 
6 month 0.80115366 0.800128 -0.001120179 -.00022403
PROBLEM 
 The direct quote in Tokyo for Peso is given as 
¥28.8358/MP Bid & ¥ 28.8725/MP Ask, in México city 
0.04418/0.04488. 
 Calculate the bid –ask Spread as percentage of bid price 
from the Japanese and from the Mexican perspective. 
 would there exist an opportunity for profitable arbitrage? 
If yes describe the necessary transaction assuming a ¥ 1 
million starting amount.
SOLUTION 
Direct terms Bid rate Ask rate 
In Japan ¥ 28.8358/MP ¥28.8725/MP 
In Mexico MP 0.04418/¥ MP 0.04422/¥ 
 Bid-Ask- spread as a % in yen (Japanese) = (28.8725- 
28.8358)/ 28.8358 = 0.127%. 
 Bid –Ask-spread as % in MP (Mexican) = (0.04422- 
0.04418) / 0.04418 = -0.09%. 
Opportunities for arbitrage:( in Mexico & in Tokyo) 
 In Mexico= MP 0.04418/ ¥ Bid: MP 0.04422/ ¥. 
Or 
¥22.6142/NP bid: ¥22.6346/MP ask
Purchase MP in Mexico at ¥ 22.6346/MP (ask) 
Assuming ¥ 1 million as the starting amount & 
converting it to MP. so, MP received = 1m / 
22.6346= 42625.7 MP. 
 In Tokyo, = ¥28.8358/MP bid and ¥28.8725/MP ask, 
convert MP into yen in Tokyo at ¥ 28.8358/MP bid. 
 Yen received = ¥1.229m 
 Profit = ¥ 1.229m- ¥ 1m = ¥.229m 
 Hence profit earned = ¥ 0.229m
PROBLEM 
 call option on euro is written with a strike price of $ 
0.9400/e at premium of 0.9000$ per euro/E and with an 
expiration data 3 month s from now . The option is for E 
100,000. calculate your profit or loss if the excise before 
maturity at the time when the euro is traded spot at: 
 $ 0.9000/e 
 $ 0.9200/e 
 $ 0.9400/e 
 $ 0.9600/e 
 $ 0.9800/e 
 $ 1.0000/e 
 $ 1.0200/e
SOL :CALL OPTION: FSP>SP =EXECUTE, PUT OPTION: FSP<SP = EXECUTE 
Spot rate Strike 
price 
Premium Decision Profit = spot 
rate – (strike 
price +Premium) 
$0.9000/e 0.9400/e 0.0090/e Execute option profit 0.049 
$0.9200/e ,, ,, Execute option profit 0.029 
$0.9400/e ,, ,, Execute option profit 0.009 
$0.9600/e ,, ,, Not Execute 0.011 
$0.9800/e ,, ,, Not Execute 0.031 
$1.000/e ,, ,, Not Execute 0.051 
$1.0200/e ,, ,, Not Execute 0.071
PROBLEM 
 Spot and 180 day forward exchange rates of 
several major currencies are given . For each pair 
calculate the percentage premium or discount 
expressed as annual rate. 
Countries Quotation spot rate 180 days forward rate 
European euro $ 0.8000/e $0.8160/e 
British pound $ 1.562/₤ $1.5300/₤ 
Japanese yen ¥120.00/$ ¥118.00/$ 
Swiss franc SF 1.6000/$ SF 1.6200/$ 
Hong kong dollar HK $ 8.000/$ HK $7.8000/$
SOLUTION 
 Forward premium/ discount = 
= FR-SR/SR X100 X360/number of days. 
 European Euro = 0.8160- 0.8000/0.8000 X 360/180X100 
Premium = 4% 
 British Pound =1.5300-1.562/1.562 X 360/180 x 100 
Discount = -4.08% 
 Japanese yen = 118.00-120.00/120.00 x360/180 x 100 
Discount = -3.33%. 
 Swiss franc = 1.6200- 1.6000/1.6000 x 360/180 x 100 
Premium = 2.5% 
 Honk kong dollar = 7.8000-8.000/8.000 x 360/180 x 100 
Discount= -5%
ASSIGNMENT PROBLEMS
PROBLEM:1 
 Given the following date calculate the arbitrage 
possibilities 
Spot rate 1 $ = Rs 42.0010. 
6 month forward rate 1$ = Rs 42.8020 
Annualised interest rate on 6 month Rs:12 % 
Annualised interest rate on 6 month USD:8% 
Is arbitrage is possible.
PROBLEM:2 
 Given the following data: 
Spot rate = 46.0010 Rs =1$ 
6 month forward rate Rs 46.8020 =1$ 
Annualised interest rate on 6 month Rs = 12% 
Annualised interest rate on 6 month $ =8% 
Initial investment is 10,000$ 
Calculate the arbitrage possibilities.
PROBLEM:3 
 USD/INR spot rate = 53.75/90. 2 month swap =.12/.20 
 USD/JPY spot rate = 110.50/125.45 . 2 month swap 
=.20/.15 
 Find INR/JPY , 2 month outright
PROBLEM: 4 
 The current CHF/USD spot =0.6675. the following 90 day 
call option on CHF is available. 
Strike price Premium 
0.60 0.075 
0.65 0.03 
0.68 0.01 
0.70 0.005 
0.75 0.002 
Your view is that CHF is going to make strong up move 
during the next 90 days your risk appetite is moderate . 
What strategy is suitable for you/ explain with the pay off 
table
EXOSURE : 
TRANSLATION , TRANSACTION, 
OPERTING
PROBLEM: 
 Farm product is the Canadian affiliate of a US 
manufacturing company .its balance sheet in thousand 
of Canadian $ for dollars for January, the January 1 
20x2, exchange rate was C $ 1.6/$. 
Farm product balance sheet (thousand of C$) 
Assets Liabilities and networth 
Cash =C$ 100000 
Account receivable = 220000 
Inventory = 320000 
Net plant and equipment = 200000 
--------------- 
Total 8,40,000 
Current liabilities =C $ 60,000 
Long term debt = 160,000 
Capital stock =620000 
---------------- 
total 8,40,000
 Determine farm product accounting exposure on 
January 1 2012, using the current rate method/ 
monetary /non monetary method. 
 Calculate farm product contribution to its parent 
accounting loss if the exchange rate on December 
3 20x2 was C$ 1.8 per $. Assume all accounts 
remain as they were at the beginning of the year.
SOLUTION 
Current /non 
current method 
C$ Exchange 
rate 
Conversion to US$ on jan 1 
20x2 
Assets 
1. Account 
receivable 
2. cash 
3. Inventory 
4. Plant & 
equipment 
Liabilities 
1. Current 
liabilities 
2. Long term 
debt 
3. Capital stock 
CTA 
100,000 
220,000 
320,000 
200,000 
60,000 
160,000 
620,000 
======== 
840,000 
1.8 
1.8 
1.8 
1.8 
1.8 
1.8 
1.6 
(100000/1.8) 
= 55,555.55 
= 122,222.22 
= 1,77,777.77 
= 1,11,111.11 
-------------------- 
= 4,66,666.66 
------------------- 
= 33,333.33 
= 88,888.88 
= 3,87,500.00 
= (43,055.55) 
-------------------- 
4,66,666.66
As per current rate method 
Accounting exposure on Jan 1 20x2 is as follows 
Exposed asset = US$ 4,66,666.66. 
Exposed liabilities = US $ 1,22, 222.22. 
Accounting exposure = 3,44,444.45 
Accounting loss as show as in CTA (Cumulative Translation 
Adjustment) account is =US $ 43,055.55 
 Cumulative Translation Adjustment :- Cumulative Translation 
Adjustments are an integral part of the financial statements for firms 
with international market exposure. 
 An entry in the comprehensive income section of a translated 
balance sheet summarizing the gains/losses resulting from varying 
exchange rates over the years. A CTA entry is required under the 
Financial Accounting Standards Board (FASB) No.52 rule as a 
means of helping investors differentiate between actual operating 
gains/losses and those generated via translation. 
 
(Opening Functional Balance x (Current month-end rate - Previous 
month-end rate)] + [Sum(Transactions in month) x (Current month-end 
rate - current average rate)] 
) x [-1]
Monetary/non 
monetary method 
C$ Exchange rate Cinversion to US 
$ on jan 1 20x2 
• Cash 
• Account 
receivable 
• Inventory 
• Plant & equipment 
• =========== 
• Current liabilities 
• Long term debt 
• Capital stock 
CTA 
100,000 
220,000 
320,000 
200,000 
========= 
60,000 
160,000 
620,000 
1.8 
1.8 
1.6 
1.6 
1.8 
1.8 
1.6 
55,555.55 
122,222.22 
200,000.00 
125,000 
========== 
5,02,777.77 
33,333.33 
88,888.88 
38,7500. 
(6,944.44)
AS PER THE MONETARY/ NON MONETARY 
METHOD 
 Accounting exposure on jan /1/20x2 
Exposure assets= US $ 1,77,777.77 
Exposure liabilities =US $ 1,22,222.21 
Accounting exposure = 55555.57 
Accounting lose as on CTA a/c =US $ 6,944.44
PROBLEM 
ABC house ltd manufactures orange marmalade 
in England .it is the wholly owned subsidiary of 
XYZ Inc. of USA the financial currency for ABC 
is the pound sterling which currently sells at 
$1.5000/£. The reporting currency for XYZ is the 
US $ . Non- consolidated financial statement for 
both ABC and XYZ are as follows ( in thousand )
ASSETS XYZ ABC LIBILITES XYZ ABC 
• CASH 
• ACCOUNT 
RECEIVABLE 
• INVENTORY 
• NET PLANT & 
EQUIPMENT 
• INVESTMENT 
$ 8000 
10,000 
8,000 
10,000 
4,500 
₤ 2000 
4000 
2000 
6000 
---- 
• CURRENT 
LIABILTIES 
• 5 YEAR TERM 
LOAN 
• CAPITAL STOCK 
• RETAINED 
EARNING 
$ 22,000 
---- 
9,000 
9,500 
£ 4,000 
4,000 
2,000 
4,000 
Prepare a consolidation balance sheet for XYZ ltd. 
What is ABC LTD accounting exposure in dollars ? Use the current rate 
method of calculation. 
Before n business activates take place the pound sterling deprecation 9% in 
value relative to the dollars what is the new spot rate? 
What is XYZ accounting loss or gain if any by the current rate method / 
Monterey non monetary method
SOLUTION 
 Balance sheet for ABC ltd in dollar 
Assets ABC Rate $ Liabilities ABC Rate $ 
CASH ₤2000 1.5 3000 CURRENT 
LIABILTIES 
£ 4000 1.5 6000 
ACCOUNT 
RECEIVABLE 
4000 1.5 6000 5 YEAR TERM 
LOAN 
4000 1.5 6000 
INVENTORY 2000 1.5 3000 CAPITAL 
STOCK 
2000 1.5 3000 
NET PLANT & 
EQUIPMENT 
6000 1.5 9000 RETAINED 
EARNING 
4000 1.5 6000 
14000 21000 14000 21000
CONSOLIDATION BALANCE SHEET FOR XYZ 
AND ITS SUBSIDIARY ABC LTD 
Assets Amount Liabilties Amount 
CASH $11000 
(8000+3000) 
Current 
liabilities 
$ 28000 
(22000+6000) 
ACCOUNT 
RECEIVABLE 
16000 
(10000+6000) 
5 year term loan 6000 (0+ 6000) 
INVENTORY 11000 
(8000+3000) 
Capital stock 12000 
(9000+3000) 
NET PLANT & 
EQUIPMENT 
19000(10000+9 
000) 
Retained 
earning 
15500 
(9500+6000) 
INVESTMENT 4500 
Total 61500 Total 61500
B. USING THE CURRENT RATE METHOD 
Assets Amount 
Cash $ 3000 
Accounts receivable $ 6000 
Inventory $ 3000 
Net plant & equipment $ 9000 
total $ 21000 
Liabilities tterm loan Amount 
Current liabilities $ 6000 
5 yrs term loan 6000 
Capital stock 2000 
Retained earning 4000 
total 18000
CONTI……………. 
 Accounting Exposure (21000-18000) = $3000. 
 New position of the firm after deprecation 
Assets Current rate 
method 
m/nm amount 
Cash $ 2730 
(3000*9%)-3000 
$ 2730 
Account 
receivable 
$ 5460 
(6000*9%)-6000 
$ 5460 
Inventory $ 2730 
(3000*9%)-3000 
$ 3000 
Net plant 
equipment 
$ 8190 
(9000*9%)-9000 
$ 9000 
Total $ 19110 $ 20190
Liabilities & net worth 
Current liabilities $ 5460 $ 5460 
5 yrs term loan $ 5460 $ 5460 
Capital stock $ 3000 $ 3000 
Retained earning $ 6000 $ 6000 
CTA (810) 270 
total $19110 $ 20190 
Loss by current method =$ 810 
Gain monetary/non monetary method =270$
PROBLEM 
 New Haven a dealer based in Europe is owned by 
an MNC inc. of the united states. Given below is 
new haven’s balance sheet at the current exchange 
rate of $1.50/ euro. 
Value in euro At value at $ 1.5/euro 
Assets 
Cash & short term securities $ 50,000 $ 75,000 
Account receivable $ 30,000 $ 45,000 
Inventory $ 20,000 $ 30,000 
Plant equipment $ 600,000 $ 900,000 
Total assets $700,000 $10,50,000
Liabilities Value in euro Value at $1.5/e 
Account payable $ 150,000 $ 225,000 
Short term debt $ 60,000 $ 90,000 
Long term debt $ 410,000 $ 615,000 
Net worth $ 80,000 $ 120,000 
Total liabilities and net 
$7,00,000 $ 10,50,000 
worth 
For the current /non current method the temporal method and the all current 
rate method calculate . 
a. The company’s exposed assets exposed liabilities and net exposed assets 
under each accounting translation method 
b. Suppose the euro depreciate by25% identify the impact of 25% 
depreciation of the euro on new haven’s consolidation balance sheet under 
each accounting translation method.
SOLUTION 
Value in 
euro 
Value at 1.50 (in 
$) 
Assets; 
• Cash and short term 
securities 
• Account receivable 
• Inventory 
• Plant and equipment 
Total assets 
Liabilities 
• Account receivable 
• Short term debt 
• Long term debt 
• Net worth 
Total liabilities & net 
worth 
50,000 
30,000 
20,000 
600,000 
700,000 
150000 
60,000 
410000 
80,000 
700,000 
75000 
45000 
30000 
900,000 
1050000 
2,25000 
90,000 
615,000 
120,000 
1050,000
Deprecation = 25% 
initial euro = 1.5$ 
that is 1 $ = .67 euro, Deprecation of 25% implies 1 $ = 
1.25 x .67 euro =.83 
which is same as 1 euro = 1.2 $. 
All current method: 
 Expose assets: 8,40,000$ 
 Expose liabilities : 744,000$ 
 Net exposed assets: 96,000$ 
Current /non current method: 
 Expose assets : 120,000$ 
 Expose liabilities: 252000$ 
 Net exposed asset: =132,000$ 
Temporal method: 
 Exposed assets: 120,000$ 
 Exposed liabilities: 744,000$ 
 Net exposed assets: = -624,000$
Balance sheet In euro Ex 
. 
rat 
e 
C /non C ($) Ex. 
rate 
Tempora 
l ($) 
Ex. 
rate 
Cur. Rate 
($) 
Assets: 
• Cash & short 
term 
securities. 
• Account 
receivable 
• Inventory 
• Plant & 
equipment 
Total asset 
Liabilities 
• Account 
payable 
• Short term 
securities 
• Long term 
securities 
• Net worth 
Total liabilities 
Translation 
exposure 
50,000 
30,000 
20,000 
600,00 
700,000 
150,000 
60,000 
410,000 
80,000 
700,000 
1.2 
1.2 
1.2 
1.5 
1.2 
1.2 
1.5 
1.5 
(50,000x1.2) 
60,000 
36000 
24000 
900,000 
1020,000 
180000 
72000 
615000 
120000 
987000 
33000 
1.2 
1.2 
1.2 
1.5 
1.2 
1.2 
1.2 
1.5 
60,000 
36000 
24000 
900,000 
1020,000 
180,000 
72,000 
492,000 
120,000 
864000 
156000 
1.2 
1.2 
1.2 
1.2 
1.2 
1.2 
1.2 
1.5 
60,000 
36000 
24000 
720,000 
840,000 
180000 
72000 
492000 
120000 
864000 
-24000
PROBLEM 
 An MNC has account receivable of $ 1.8 billion and 
account payable of $940 million . It also has 
borrowed $ 700 million . The current spot rate is $ 
1.81138/£ 
 What is the MNC dollar transaction exposure in 
dollar terms? In pound term? 
 Suppose the pound appreciates to $ 2.1122 /₤ what 
is the MNC gain or loss in pound terms , on its 
dollar transaction exposure?
SOLUTION 
In dollar term: 
Functional currency = dollar (i.e. foreign currency so the 
method used would be current rate method) 
Net exposure = exposure asset – exposure liabilities. 
Current method 
Exposure assets = $ 1800 mn 
Exposure liabilities = $ 940+ $700= $ 1640 
Translation exposure = 1800-1640 = $ 160 mn
 In pound term 
Translation exposure = 1800- 1640 = $ 160 mn 
Current rate = 1.8138/£ 
so translation exposure = 160/1.8138 = ₤ 88.21mn 
b. The pound appreciation to $ 2.1122/£ 
Net exposure = ₤ 160 mn 
Current rate = $ 2.1122/£ 
so translation exposure = 160/2.1122 = ₤ 75.75 
So this is a loss of (£ 88.21 – £ 75.75) = £12.46 mn
PROBLEM 
A foreign company expect to receive Mexico $ 15 
million entertainment fees from Mexico in 90 days . 
The current spot rate is $0.2320/M$ and the 90 day 
forward rate is 0.2240/M$. 
 What I the company’s peso transaction exposure 
associated with this fee? 
 What is the expected US $ value of the fee if the 
spot rate expected in 90 days in $0.2305. also 
calculate the hedged dollar value of the fee?
SOLUTION 
 Mex $ received if the fee to be paid today = $ million 
15 x .2320 = US $ 3.48 million 
 Mex $ received by the foreign company if the fee is to be 
paid in 90 days = 15 x .2240 = US $ 3.36 million 
 Transaction exposure = 3.36 - 3.48 = -0.12 million 
B. The expected US$ fee = 15 x .2305 = Us $ 34,57,500 
 The hedged dollar value of the fee is = 3480,000 – 
3457000 = $ 23,000.
PROBLEM 
 AV ltd is the Indian affiliate of affiliate of a US 
sports manufacture . AV ltd manufacture items 
which are sold primarily in the united states and 
Europe .AV balance sheet in thousand of rupees as 
of march 31 is as follows: 
Assets Liabilities 
Cash $ 6000 
Account receivable $ 4500 
Inventory $ 4500 
Net plant equipment $ 10,000 
Total $ 25000 
Account payable $ 3500 
Short term bank loan $ 1500 
Long term loan $ 4000 
Capital stock $ 10,000 
Retained earning $ 6000
Exchange rate for translating the balance sheet into 
US $ are: 
 RS 35/$ : historic exchange rate , at which plant 
and equipment long term loan and common stock 
were acquired or issued. 
 RS 40/$ : march 31 exchange rate this was also the 
rate at which inventory was acquired. 
 RS 42/$ April 1 exchange rate , after devaluation of 
20%. 
 Assuming no change in balance sheet a/c between 
march 31 and April 1 calculate accounting gain 
/loss by the current rate method and by monetary / 
non monetary method explain accounting loss in 
term of change in the value of exposed accounts
SOLUTION 
Exchange rate: 
 RS 35/$ historical rate for the others 
 RS 40/$ for inventory march 31 – historical rate for 
inventory. 
 RS 42/$ for April 1 – current rate as on April 1
Rs Ex. 
Change 
rate 
Conversion to $ 
on Nov 31 
Ex. 
Change 
rate 
Conversion to 
$ on April 1 
• Cash 
• A/c 
Receivable 
• Inventory 
• Plant & 
equipment 
• Total 
• Account 
payable 
• short term 
loan 
• Long term 
loan 
• Capital 
stock 
• retained 
earning 
• CTA 
• Total 
6000 
4500 
4500 
10,000 
25000 
3500 
1500 
4000 
10,000 
6000 
40 
40 
40 
40 
40 
40 
40 
35 
35 
$ 150 
$112.50 
$112.50 
$250.00 
$625.00 
$87.50 
$37.50 
$100.00 
$285.71 
$171.43 
(57.14) 
$625.00 
42 
42 
42 
42 
42 
42 
42 
35 
35 
$ 142.50 
$107.14 
$107.14 
$238.10 
$595.24 
$83.33 
$35.71 
$95.24 
$285.71 
$171.043 
(76.19) 
$595.24
ACCOUNTING LOSS BY CURRENT RATE 
METHOD IS $ 57.14 ON MARCH 31 
March 31 April 1 
Exposed assets $ 625.00 $ 595.24 
Exposed liabilities $ 225.00 $ 214.28 
Net exposed $ 400.00 $ 380.96
Monetary /non monetary method 
Rs Ex. Change 
rate 
Conversion to $ on 
Nov 31 
Ex. Change 
rate 
Conversion to $ on 
April 1 
• Cash 
• A/c 
Receivable 
• Inventory 
• Plant & 
equipment 
• Total 
• Account 
payable 
• short term 
loan 
• Long term 
loan 
• Capital 
stock 
• retained 
earning 
• CTA 
• Total 
6000 
4500 
4500 
10,000 
25000 
3500 
1500 
4000 
10,000 
6000 
40 
40 
35 
35 
40 
40 
40 
35 
35 
$ 150 
$112.50 
$128.50 
$285.71 
$676.78 
$87.50 
$37.50 
$100.00 
$285.71 
$171.43 
(5.36) 
$676.78 
42 
42 
35 
35 
42 
42 
42 
35 
35 
$ 142.50 
$107.14 
$128.57 
$285.71 
$664.29 
$83.33 
$35.71 
$95.24 
$285.71 
$171.043 
(7.13) 
$664.29
MONETARY / NON MONETARY METHOD 
 Accounting loss by monetary / non monetary 
Is = $ 5.36 on march 31,= $ 7.13 on April 1 
March 31 April 1 
Exposed assets $ 262.50 $ 250.00 
Exposed liabilities $ 225.00 $ 214.28 
Net exposed $ (37.5) $ (35.72)
THE END

More Related Content

What's hot (20)

Spot and Forward Exchange Rate
Spot and Forward Exchange RateSpot and Forward Exchange Rate
Spot and Forward Exchange Rate
 
Interest rate parity (Global Finance)
Interest rate parity (Global Finance)Interest rate parity (Global Finance)
Interest rate parity (Global Finance)
 
Currency Exchange Risk
Currency Exchange Risk Currency Exchange Risk
Currency Exchange Risk
 
International Financial Markets
International Financial MarketsInternational Financial Markets
International Financial Markets
 
Foreign exchange risk
Foreign exchange riskForeign exchange risk
Foreign exchange risk
 
Forward and futures - An Overview
Forward and futures - An OverviewForward and futures - An Overview
Forward and futures - An Overview
 
Foreign exchange market
Foreign exchange marketForeign exchange market
Foreign exchange market
 
Derivative ppt
Derivative pptDerivative ppt
Derivative ppt
 
Forex ppt
Forex pptForex ppt
Forex ppt
 
Foreign exchange exposure PPT
Foreign exchange exposure PPTForeign exchange exposure PPT
Foreign exchange exposure PPT
 
INTERNATIONAL FINANCIAL MANAGEMENT
INTERNATIONAL FINANCIAL MANAGEMENTINTERNATIONAL FINANCIAL MANAGEMENT
INTERNATIONAL FINANCIAL MANAGEMENT
 
International bond market ppt
International  bond market   pptInternational  bond market   ppt
International bond market ppt
 
Bond immunization
Bond immunizationBond immunization
Bond immunization
 
Derivative market in india
Derivative market in indiaDerivative market in india
Derivative market in india
 
FIM - Currency Futures PPT
FIM - Currency Futures PPTFIM - Currency Futures PPT
FIM - Currency Futures PPT
 
Modern Portfolio Theory
Modern Portfolio TheoryModern Portfolio Theory
Modern Portfolio Theory
 
Forex Market
Forex MarketForex Market
Forex Market
 
Spot market
Spot marketSpot market
Spot market
 
Financial derivatives ppt
Financial derivatives pptFinancial derivatives ppt
Financial derivatives ppt
 
Participants in foreign exchange market
Participants in foreign  exchange marketParticipants in foreign  exchange market
Participants in foreign exchange market
 

Similar to Ifm problems

Numericals on IF.pptx
Numericals on IF.pptxNumericals on IF.pptx
Numericals on IF.pptxPravalSantar1
 
Forex_Hedging (1).pptx
Forex_Hedging (1).pptxForex_Hedging (1).pptx
Forex_Hedging (1).pptxssuser2c87431
 
Foreign exchange Risk [Autosaved].pptx
Foreign exchange Risk [Autosaved].pptxForeign exchange Risk [Autosaved].pptx
Foreign exchange Risk [Autosaved].pptxPravalSantar1
 
Bfm numericals (all mods)
Bfm numericals (all mods)Bfm numericals (all mods)
Bfm numericals (all mods)Vinayak Kamath
 
The Time Value of Money
The Time Value of MoneyThe Time Value of Money
The Time Value of MoneyJyoti Yadav
 
Time value of money
Time value of moneyTime value of money
Time value of moneyUsman Nazir
 
4th Lecture- discounted cash flows (1).pptx
4th Lecture- discounted cash flows (1).pptx4th Lecture- discounted cash flows (1).pptx
4th Lecture- discounted cash flows (1).pptxabdelhameedibrahim4
 
Jm 4th edition_solutions_manual
Jm 4th edition_solutions_manualJm 4th edition_solutions_manual
Jm 4th edition_solutions_manualNoor Ahammad
 
Fixed income bonds
Fixed income bondsFixed income bonds
Fixed income bondsSapna Desai
 
Business Finance Chapter 7
Business Finance Chapter 7Business Finance Chapter 7
Business Finance Chapter 7Tinku Kumar
 
Aminullah assagaf p610 ch. 6 sd10_financial management_28 mei 2021
Aminullah assagaf p610 ch. 6 sd10_financial management_28 mei 2021Aminullah assagaf p610 ch. 6 sd10_financial management_28 mei 2021
Aminullah assagaf p610 ch. 6 sd10_financial management_28 mei 2021Aminullah Assagaf
 
Aminullah assagaf financial management p610_ch. 6 sd10_28 mei 2021
Aminullah assagaf financial management p610_ch. 6 sd10_28 mei 2021Aminullah assagaf financial management p610_ch. 6 sd10_28 mei 2021
Aminullah assagaf financial management p610_ch. 6 sd10_28 mei 2021Aminullah Assagaf
 
Chapter 5 Time Value of Money.pptx
Chapter 5 Time Value of Money.pptxChapter 5 Time Value of Money.pptx
Chapter 5 Time Value of Money.pptxHazlina Hussein
 
Time Value of Money- managerial finance
Time Value of Money- managerial financeTime Value of Money- managerial finance
Time Value of Money- managerial financeTanjin Tamanna urmi
 
Textual notes time value of money
Textual notes   time value of moneyTextual notes   time value of money
Textual notes time value of moneypriyameril
 
Understanding the Time Value of Money; Single Payment
Understanding the Time Value of Money; Single PaymentUnderstanding the Time Value of Money; Single Payment
Understanding the Time Value of Money; Single PaymentDIANN MOORMAN
 

Similar to Ifm problems (20)

Numericals on IF.pptx
Numericals on IF.pptxNumericals on IF.pptx
Numericals on IF.pptx
 
Chap005
Chap005Chap005
Chap005
 
Forex_Hedging (1).pptx
Forex_Hedging (1).pptxForex_Hedging (1).pptx
Forex_Hedging (1).pptx
 
Foreign exchange Risk [Autosaved].pptx
Foreign exchange Risk [Autosaved].pptxForeign exchange Risk [Autosaved].pptx
Foreign exchange Risk [Autosaved].pptx
 
Bfm numericals (all mods)
Bfm numericals (all mods)Bfm numericals (all mods)
Bfm numericals (all mods)
 
The Time Value of Money
The Time Value of MoneyThe Time Value of Money
The Time Value of Money
 
Time value of money
Time value of moneyTime value of money
Time value of money
 
Chap022
Chap022Chap022
Chap022
 
4th Lecture- discounted cash flows (1).pptx
4th Lecture- discounted cash flows (1).pptx4th Lecture- discounted cash flows (1).pptx
4th Lecture- discounted cash flows (1).pptx
 
Jm 4th edition_solutions_manual
Jm 4th edition_solutions_manualJm 4th edition_solutions_manual
Jm 4th edition_solutions_manual
 
Fixed income bonds
Fixed income bondsFixed income bonds
Fixed income bonds
 
Foreign exchange
Foreign exchangeForeign exchange
Foreign exchange
 
Business Finance Chapter 7
Business Finance Chapter 7Business Finance Chapter 7
Business Finance Chapter 7
 
Aminullah assagaf p610 ch. 6 sd10_financial management_28 mei 2021
Aminullah assagaf p610 ch. 6 sd10_financial management_28 mei 2021Aminullah assagaf p610 ch. 6 sd10_financial management_28 mei 2021
Aminullah assagaf p610 ch. 6 sd10_financial management_28 mei 2021
 
Aminullah assagaf financial management p610_ch. 6 sd10_28 mei 2021
Aminullah assagaf financial management p610_ch. 6 sd10_28 mei 2021Aminullah assagaf financial management p610_ch. 6 sd10_28 mei 2021
Aminullah assagaf financial management p610_ch. 6 sd10_28 mei 2021
 
Chapter 5 Time Value of Money.pptx
Chapter 5 Time Value of Money.pptxChapter 5 Time Value of Money.pptx
Chapter 5 Time Value of Money.pptx
 
Time Value of Money- managerial finance
Time Value of Money- managerial financeTime Value of Money- managerial finance
Time Value of Money- managerial finance
 
Textual notes time value of money
Textual notes   time value of moneyTextual notes   time value of money
Textual notes time value of money
 
Understanding the Time Value of Money; Single Payment
Understanding the Time Value of Money; Single PaymentUnderstanding the Time Value of Money; Single Payment
Understanding the Time Value of Money; Single Payment
 
Discounted cash flow valuation
Discounted cash flow valuationDiscounted cash flow valuation
Discounted cash flow valuation
 

More from Acharya Bangalore Business School (11)

international financial management
international financial managementinternational financial management
international financial management
 
financial institution and markets
financial institution and marketsfinancial institution and markets
financial institution and markets
 
financial markets
financial marketsfinancial markets
financial markets
 
Foreign exchange markets
Foreign exchange markets Foreign exchange markets
Foreign exchange markets
 
Money markets
Money marketsMoney markets
Money markets
 
New term in issue markets
New term in issue marketsNew term in issue markets
New term in issue markets
 
Primary markets eligibility norms
Primary markets eligibility normsPrimary markets eligibility norms
Primary markets eligibility norms
 
Underwriters to bankers
Underwriters to bankersUnderwriters to bankers
Underwriters to bankers
 
Underwriters to bankers
Underwriters to bankersUnderwriters to bankers
Underwriters to bankers
 
Lagaan ppt
Lagaan pptLagaan ppt
Lagaan ppt
 
Merchant bankers
Merchant bankersMerchant bankers
Merchant bankers
 

Recently uploaded

The Economic History of the U.S. Lecture 19.pdf
The Economic History of the U.S. Lecture 19.pdfThe Economic History of the U.S. Lecture 19.pdf
The Economic History of the U.S. Lecture 19.pdfGale Pooley
 
Independent Call Girl Number in Kurla Mumbai📲 Pooja Nehwal 9892124323 💞 Full ...
Independent Call Girl Number in Kurla Mumbai📲 Pooja Nehwal 9892124323 💞 Full ...Independent Call Girl Number in Kurla Mumbai📲 Pooja Nehwal 9892124323 💞 Full ...
Independent Call Girl Number in Kurla Mumbai📲 Pooja Nehwal 9892124323 💞 Full ...Pooja Nehwal
 
Lundin Gold April 2024 Corporate Presentation v4.pdf
Lundin Gold April 2024 Corporate Presentation v4.pdfLundin Gold April 2024 Corporate Presentation v4.pdf
Lundin Gold April 2024 Corporate Presentation v4.pdfAdnet Communications
 
VIP Call Girls Service Dilsukhnagar Hyderabad Call +91-8250192130
VIP Call Girls Service Dilsukhnagar Hyderabad Call +91-8250192130VIP Call Girls Service Dilsukhnagar Hyderabad Call +91-8250192130
VIP Call Girls Service Dilsukhnagar Hyderabad Call +91-8250192130Suhani Kapoor
 
Solution Manual for Financial Accounting, 11th Edition by Robert Libby, Patri...
Solution Manual for Financial Accounting, 11th Edition by Robert Libby, Patri...Solution Manual for Financial Accounting, 11th Edition by Robert Libby, Patri...
Solution Manual for Financial Accounting, 11th Edition by Robert Libby, Patri...ssifa0344
 
Interimreport1 January–31 March2024 Elo Mutual Pension Insurance Company
Interimreport1 January–31 March2024 Elo Mutual Pension Insurance CompanyInterimreport1 January–31 March2024 Elo Mutual Pension Insurance Company
Interimreport1 January–31 March2024 Elo Mutual Pension Insurance CompanyTyöeläkeyhtiö Elo
 
20240417-Calibre-April-2024-Investor-Presentation.pdf
20240417-Calibre-April-2024-Investor-Presentation.pdf20240417-Calibre-April-2024-Investor-Presentation.pdf
20240417-Calibre-April-2024-Investor-Presentation.pdfAdnet Communications
 
Log your LOA pain with Pension Lab's brilliant campaign
Log your LOA pain with Pension Lab's brilliant campaignLog your LOA pain with Pension Lab's brilliant campaign
Log your LOA pain with Pension Lab's brilliant campaignHenry Tapper
 
call girls in Nand Nagri (DELHI) 🔝 >༒9953330565🔝 genuine Escort Service 🔝✔️✔️
call girls in  Nand Nagri (DELHI) 🔝 >༒9953330565🔝 genuine Escort Service 🔝✔️✔️call girls in  Nand Nagri (DELHI) 🔝 >༒9953330565🔝 genuine Escort Service 🔝✔️✔️
call girls in Nand Nagri (DELHI) 🔝 >༒9953330565🔝 genuine Escort Service 🔝✔️✔️9953056974 Low Rate Call Girls In Saket, Delhi NCR
 
Call US 📞 9892124323 ✅ Kurla Call Girls In Kurla ( Mumbai ) secure service
Call US 📞 9892124323 ✅ Kurla Call Girls In Kurla ( Mumbai ) secure serviceCall US 📞 9892124323 ✅ Kurla Call Girls In Kurla ( Mumbai ) secure service
Call US 📞 9892124323 ✅ Kurla Call Girls In Kurla ( Mumbai ) secure servicePooja Nehwal
 
VVIP Pune Call Girls Katraj (7001035870) Pune Escorts Nearby with Complete Sa...
VVIP Pune Call Girls Katraj (7001035870) Pune Escorts Nearby with Complete Sa...VVIP Pune Call Girls Katraj (7001035870) Pune Escorts Nearby with Complete Sa...
VVIP Pune Call Girls Katraj (7001035870) Pune Escorts Nearby with Complete Sa...Call Girls in Nagpur High Profile
 
VIP Call Girls LB Nagar ( Hyderabad ) Phone 8250192130 | ₹5k To 25k With Room...
VIP Call Girls LB Nagar ( Hyderabad ) Phone 8250192130 | ₹5k To 25k With Room...VIP Call Girls LB Nagar ( Hyderabad ) Phone 8250192130 | ₹5k To 25k With Room...
VIP Call Girls LB Nagar ( Hyderabad ) Phone 8250192130 | ₹5k To 25k With Room...Suhani Kapoor
 
00_Main ppt_MeetupDORA&CyberSecurity.pptx
00_Main ppt_MeetupDORA&CyberSecurity.pptx00_Main ppt_MeetupDORA&CyberSecurity.pptx
00_Main ppt_MeetupDORA&CyberSecurity.pptxFinTech Belgium
 
Andheri Call Girls In 9825968104 Mumbai Hot Models
Andheri Call Girls In 9825968104 Mumbai Hot ModelsAndheri Call Girls In 9825968104 Mumbai Hot Models
Andheri Call Girls In 9825968104 Mumbai Hot Modelshematsharma006
 
Call Girls In Yusuf Sarai Women Seeking Men 9654467111
Call Girls In Yusuf Sarai Women Seeking Men 9654467111Call Girls In Yusuf Sarai Women Seeking Men 9654467111
Call Girls In Yusuf Sarai Women Seeking Men 9654467111Sapana Sha
 
(ANIKA) Budhwar Peth Call Girls Just Call 7001035870 [ Cash on Delivery ] Pun...
(ANIKA) Budhwar Peth Call Girls Just Call 7001035870 [ Cash on Delivery ] Pun...(ANIKA) Budhwar Peth Call Girls Just Call 7001035870 [ Cash on Delivery ] Pun...
(ANIKA) Budhwar Peth Call Girls Just Call 7001035870 [ Cash on Delivery ] Pun...ranjana rawat
 
High Class Call Girls Nagpur Grishma Call 7001035870 Meet With Nagpur Escorts
High Class Call Girls Nagpur Grishma Call 7001035870 Meet With Nagpur EscortsHigh Class Call Girls Nagpur Grishma Call 7001035870 Meet With Nagpur Escorts
High Class Call Girls Nagpur Grishma Call 7001035870 Meet With Nagpur Escortsranjana rawat
 
VIP Kolkata Call Girl Serampore 👉 8250192130 Available With Room
VIP Kolkata Call Girl Serampore 👉 8250192130  Available With RoomVIP Kolkata Call Girl Serampore 👉 8250192130  Available With Room
VIP Kolkata Call Girl Serampore 👉 8250192130 Available With Roomdivyansh0kumar0
 
Instant Issue Debit Cards - School Designs
Instant Issue Debit Cards - School DesignsInstant Issue Debit Cards - School Designs
Instant Issue Debit Cards - School Designsegoetzinger
 

Recently uploaded (20)

The Economic History of the U.S. Lecture 19.pdf
The Economic History of the U.S. Lecture 19.pdfThe Economic History of the U.S. Lecture 19.pdf
The Economic History of the U.S. Lecture 19.pdf
 
Commercial Bank Economic Capsule - April 2024
Commercial Bank Economic Capsule - April 2024Commercial Bank Economic Capsule - April 2024
Commercial Bank Economic Capsule - April 2024
 
Independent Call Girl Number in Kurla Mumbai📲 Pooja Nehwal 9892124323 💞 Full ...
Independent Call Girl Number in Kurla Mumbai📲 Pooja Nehwal 9892124323 💞 Full ...Independent Call Girl Number in Kurla Mumbai📲 Pooja Nehwal 9892124323 💞 Full ...
Independent Call Girl Number in Kurla Mumbai📲 Pooja Nehwal 9892124323 💞 Full ...
 
Lundin Gold April 2024 Corporate Presentation v4.pdf
Lundin Gold April 2024 Corporate Presentation v4.pdfLundin Gold April 2024 Corporate Presentation v4.pdf
Lundin Gold April 2024 Corporate Presentation v4.pdf
 
VIP Call Girls Service Dilsukhnagar Hyderabad Call +91-8250192130
VIP Call Girls Service Dilsukhnagar Hyderabad Call +91-8250192130VIP Call Girls Service Dilsukhnagar Hyderabad Call +91-8250192130
VIP Call Girls Service Dilsukhnagar Hyderabad Call +91-8250192130
 
Solution Manual for Financial Accounting, 11th Edition by Robert Libby, Patri...
Solution Manual for Financial Accounting, 11th Edition by Robert Libby, Patri...Solution Manual for Financial Accounting, 11th Edition by Robert Libby, Patri...
Solution Manual for Financial Accounting, 11th Edition by Robert Libby, Patri...
 
Interimreport1 January–31 March2024 Elo Mutual Pension Insurance Company
Interimreport1 January–31 March2024 Elo Mutual Pension Insurance CompanyInterimreport1 January–31 March2024 Elo Mutual Pension Insurance Company
Interimreport1 January–31 March2024 Elo Mutual Pension Insurance Company
 
20240417-Calibre-April-2024-Investor-Presentation.pdf
20240417-Calibre-April-2024-Investor-Presentation.pdf20240417-Calibre-April-2024-Investor-Presentation.pdf
20240417-Calibre-April-2024-Investor-Presentation.pdf
 
Log your LOA pain with Pension Lab's brilliant campaign
Log your LOA pain with Pension Lab's brilliant campaignLog your LOA pain with Pension Lab's brilliant campaign
Log your LOA pain with Pension Lab's brilliant campaign
 
call girls in Nand Nagri (DELHI) 🔝 >༒9953330565🔝 genuine Escort Service 🔝✔️✔️
call girls in  Nand Nagri (DELHI) 🔝 >༒9953330565🔝 genuine Escort Service 🔝✔️✔️call girls in  Nand Nagri (DELHI) 🔝 >༒9953330565🔝 genuine Escort Service 🔝✔️✔️
call girls in Nand Nagri (DELHI) 🔝 >༒9953330565🔝 genuine Escort Service 🔝✔️✔️
 
Call US 📞 9892124323 ✅ Kurla Call Girls In Kurla ( Mumbai ) secure service
Call US 📞 9892124323 ✅ Kurla Call Girls In Kurla ( Mumbai ) secure serviceCall US 📞 9892124323 ✅ Kurla Call Girls In Kurla ( Mumbai ) secure service
Call US 📞 9892124323 ✅ Kurla Call Girls In Kurla ( Mumbai ) secure service
 
VVIP Pune Call Girls Katraj (7001035870) Pune Escorts Nearby with Complete Sa...
VVIP Pune Call Girls Katraj (7001035870) Pune Escorts Nearby with Complete Sa...VVIP Pune Call Girls Katraj (7001035870) Pune Escorts Nearby with Complete Sa...
VVIP Pune Call Girls Katraj (7001035870) Pune Escorts Nearby with Complete Sa...
 
VIP Call Girls LB Nagar ( Hyderabad ) Phone 8250192130 | ₹5k To 25k With Room...
VIP Call Girls LB Nagar ( Hyderabad ) Phone 8250192130 | ₹5k To 25k With Room...VIP Call Girls LB Nagar ( Hyderabad ) Phone 8250192130 | ₹5k To 25k With Room...
VIP Call Girls LB Nagar ( Hyderabad ) Phone 8250192130 | ₹5k To 25k With Room...
 
00_Main ppt_MeetupDORA&CyberSecurity.pptx
00_Main ppt_MeetupDORA&CyberSecurity.pptx00_Main ppt_MeetupDORA&CyberSecurity.pptx
00_Main ppt_MeetupDORA&CyberSecurity.pptx
 
Andheri Call Girls In 9825968104 Mumbai Hot Models
Andheri Call Girls In 9825968104 Mumbai Hot ModelsAndheri Call Girls In 9825968104 Mumbai Hot Models
Andheri Call Girls In 9825968104 Mumbai Hot Models
 
Call Girls In Yusuf Sarai Women Seeking Men 9654467111
Call Girls In Yusuf Sarai Women Seeking Men 9654467111Call Girls In Yusuf Sarai Women Seeking Men 9654467111
Call Girls In Yusuf Sarai Women Seeking Men 9654467111
 
(ANIKA) Budhwar Peth Call Girls Just Call 7001035870 [ Cash on Delivery ] Pun...
(ANIKA) Budhwar Peth Call Girls Just Call 7001035870 [ Cash on Delivery ] Pun...(ANIKA) Budhwar Peth Call Girls Just Call 7001035870 [ Cash on Delivery ] Pun...
(ANIKA) Budhwar Peth Call Girls Just Call 7001035870 [ Cash on Delivery ] Pun...
 
High Class Call Girls Nagpur Grishma Call 7001035870 Meet With Nagpur Escorts
High Class Call Girls Nagpur Grishma Call 7001035870 Meet With Nagpur EscortsHigh Class Call Girls Nagpur Grishma Call 7001035870 Meet With Nagpur Escorts
High Class Call Girls Nagpur Grishma Call 7001035870 Meet With Nagpur Escorts
 
VIP Kolkata Call Girl Serampore 👉 8250192130 Available With Room
VIP Kolkata Call Girl Serampore 👉 8250192130  Available With RoomVIP Kolkata Call Girl Serampore 👉 8250192130  Available With Room
VIP Kolkata Call Girl Serampore 👉 8250192130 Available With Room
 
Instant Issue Debit Cards - School Designs
Instant Issue Debit Cards - School DesignsInstant Issue Debit Cards - School Designs
Instant Issue Debit Cards - School Designs
 

Ifm problems

  • 1. INTERNATIONAL FINANCIAL MANAGEMENT Prof. Parveen Sultana Kanth
  • 3. PROBLEM  An American investor purchased securities in Indian market investing one million dollars, at the time of investing exchange rate is 50/50.5 Rs while one year latter the exchange rate is 53/54 Rs . what is the rate of return to American investor if he rate of Rs return on Indian security is 20% 25% & 50%.
  • 4.  If the rate is = 20%  Amount invested in India = 10,00,000 * 50 = 50,000,000 (50 million) Return = 100,00,000 (1 million) After 1 year the amount to be repatriated = 60000000/54= 11,11,111.  Rate of return= 11,11,111/10,00,000 = 11.11% if the rate = 25%  Amount invested is 10,00,000 * 50 = 5,00,00, 000 at 25% return rate of return in Indian rupees = 5,00,00,000 *25% = 1,25,00,000 at the closing rate = 6,25,00,000/54 = 11,57,407 Return = 157407/10,00,000 = 15.47%
  • 5.  If the rate is 50% in Indian Rs term  Investment = 5,00,00,000  Return in Indian Rs = 5,00,00,000*50 = 2,50,00,000 Rs  Total amount to be repatriated in Rs = 7,50,00,000/54 = 13,88,889$  Return on investment = 3,88,889/10,00,000 = 38.89%
  • 6. PROBLEM :-2  Risk free return in India is 8%, an American investor wants to invest in Indian securities with beta of 2 with variance of 15%while exchange rate of rupees deprecation by 10% with variance of 20%. Market related return is 18% and correlation coefficient between return on security and exchange rate is 25% ,find out the expected rate of return and risk in investment
  • 7. CAPM THEORY IS APPLIED  As per CAPM the expected return on securities = risk free interest rate +beta (market rate –risk free interest rate). = .08 + 2 (.18 - .08) = 28% If 100 $ are invested in market then return is 128 $, then excluding exchange rate. variance of return = .15+.20+2* .25s square root of (.15* .20) = .0.44 = 44% of risk
  • 9. PROBLEM:-1 CROSS RATE  The Germany DM is selling for $ 0.62 & the buying rate for the French franc (FF) o.17$, what is the FF/DM cross rate ?  US$0.62 FF FF 3.65 ------------- x -------- = --------------- DM US$ 0.17 DM
  • 10. PROBLEM:-2 CROSS RATE  The US$ Thai Bhat exchange rate is US$ 0.02339/Bhat, and the US$ Indian Rupees is US$0.02538/INR . Suppose that INR is not quoted against Thai Bhat . What is the Bhat/INR? US$0.02538 US$ 0.02339 US$ 0.02538 Bhat ---------------- + --------------- = ------------- x --------- INR Baht US$ 0.02339 INR Ans : Baht 1.085 ---------------- INR
  • 11. THE SPOT PRICE = 0.025063,USD/INR , AND FORWARD RATE IS 0.02439, USD/INR FOR 6 MONTHS FORWARD , THE ANNUALIZED PREMIUM IS AS FOLLOWS SELLING AT PREMIUM  Forward premium spot rate- forward rate 360 Or discount = ---------------------------- x ------- forward rate days  For 6 months forward , the annualized premium is as follows 0.025063 - 0.02439 360 = -------------------------- x ------ = 5.5% 0.02439 180
  • 12.  Forward rupees is selling at a discount of 5.5%relative to the dollar for 6 months. In terms of indirect quote , the INR/$ spot exchange rate is INR 39.90/$ and forward rate is INR 41.00/$. For indirect quote, the forward premium or discount can be calculated as follows:  Forward premium Forward rate - spot rate 360 or discount = --------------------------- x ----- spot rate days 41.00 -39.90 360 = ---------------- x ------ 39.90 180 = 5.5%
  • 13. Balance of payment, Cross Rate , Spot Rate Forward Rate, exchange rate theories
  • 14. PROBLEM:-3 An exporter has to surrender $ 100,000 to bank . Rate of dollar against INR is 46/46.15, the bank charge a commission of .15%. If the transit time fixed by RBI is 20 day and the rate of interest charge by the bank is 10%.  Find out the net proceeds to be credited to exporters account?  Find out what is the amount to be paid by the person if he is an importer instead of exporter?  If Rs $ rate is 46/46.15 $/pound rate is 1.71/1.74 what is Rs pound rate.  If French franc $is 10.17/10.20 and Swiss franc $ rate is 1.73/1.75. find out French franc / Swiss franc rate?  If one Rs can buy 3.23 yen and if one Rs can buy .028 $ how many yens can 1$ buy?  In London a dealer quotes Dm/pound spot = 3.2550/3.2555,Yen pound spot = 180/181 . What do you expect yen/Dm rate in Frankfort?
  • 15.  Find out the net proceeds to be credited to exporters account? Solution: since the exporter sells the dollar to bank bid price of 46 per $ to be taken . since it involves buying of $ by bank commission should be deducted from exchange rate. Rate to be adopted is = 46 - (46 * .15%) =45.93 Total amount is 100,000* 45.93 = 45,93,000 Interest charged is 10% for 20 days on the net proceeds. = 45,93,000 * 10 *20/365 = 25,167 Rs Net proceeds to be credited to the account of exporter = 45,93,000 - 25,167 = 45,67,833
  • 16. Find out what is the amount to be paid by the person if he is an importer instead of exporter?  Solution: Since in this case the bank is selling the dollar to importer , it charges the ask rate of 46.15 and the commission is added instead of being deducted . Amount to be paid by the importer = 46.15+(Rs 46.15*.15%) = 46.22 Rs The amount to be paid by the importer to bank = $ 1,00,000* 46.22 = Rs 46,22,000.
  • 17.  If Rs $ rate is 46/46.15 $/pound rate is 1.71/1.74 what is Rs pound rate. Rs/pound rate = Rs/$* $/pound (cross rate) = 46*1.71/46.15*1.74 = 78.66/80.30  If French franc $is 10.17/10.20 and Swiss franc $ rate is 1.73/1.75. find out French franc / Swiss franc rate?  French franc /Swiss franc rate = French franc /$ * Swiss franc/$. Swiss franc /$c rate = 1/1.75/1/1.73 = .5714/.5780 French franc /Swiss franc = 10.17*.5714/10.20*.5780 = 5.81/5.90
  • 18.  If one Rs can buy 3.23 yen and if one Rs can buy .028 $ how many yens can 1$ buy? = .028 $ = 3.23 yen 1 $ = ? $ = 3.23/.028 = 115.35 yen
  • 19.  In London a dealer quotes Dm/pound spot = 3.2550/3.2555,Yen pound spot = 180/181 . What do you expect yen/Dm rate in Frankfort? Sol: Yen/Dm spot rate = yen /pound spot rate * pound /Dm spot rate. Dm/pound spot rate = 3.2550/3.2555 Pound /Dm spot rate = 1/3.2550/1/3.2555 = .3072/.3071 Yen/Dm spot rate = 181* .3072/181*.3071 = 55.603/55.278  Frankfort get quote of yen/Dm spot 51.1530/51.2550 is there an arbitrage opportunity Sol: Buy 1Dm in Frankfort by selling 51.2550 yen, sell 1 Dm and get 55.7278 yen there by making profit of 55.278 - 51.2550 = 4.023 yen
  • 20.  January , 24, 2014 a customer requested a bank to remit DG(Decigram) 250,000 to Holland as payment for Diamonds, However due to strike the bank can make payment only on Feb ,1,2014. Inter bank rates are as follows: Jan 24,2014 Feb 1 2014 Mumbai $/Rs (100Rs) 3.10/3.15 3.07/3.12 London $/pound 1.7250/60 DG /pound 3.957/90 1.7175/85 3.9380/90 How much does the customer stand to gain or lose due to delay? PROBLEM
  • 21.  Sol: Rs/Dg = Rs/$* $/Dg Rs/$ = 100/3.15/100/3.10= 31.75/32.26 = 100/3.12/100/3.07= 32.05/32.57 $/Dg =$/pound*pound/Dg = 1.7250/3.9590/1.7260/3.9575 = .4357/.4361 = 1.7175/3.9390/1.7185/3.938 = .4360/.4364 Rs /Dg = 31.75 * .4357/32.26 *.4361 = 13.83/14.07 = 32.05 *.4360/ 32.57 * .4364 = 13.97/14.21 The importer has lost (14.21 – 14.07) * 250,000 Dg = Rs 35000 due to delay remittance
  • 22. PROBLEM  Consider the value of DEM relative to USD. The spot rate is DEM 1.82. the interest rates in the US and Germany are 5% and 3% respectively. Estimate the price on 4 month forward contract on DEM.  Solution: Spot rate of Dem (1USD= DEM) 1.82 Interest rate US =5% Interest rate in Germany= 3%
  • 23. SOLUTION  Et =Eo (1+rhc/1+rfc)t. Where et = expected forward rate for time t. Eo = spot rate rhc =rate of interest in home currency. Rfc =rate of interest in foreign currency. T= time period. 1.82( 1+ 0.03/1+ 0.05) x 4//12 = 0.595
  • 24. PROBLEM USD/INR spot =48.75/80, 2 months swap =.12/.20. USD?JPY spot=125.50/126.10, 2 month swap =.20/.15. Find INR/JPY 2 month outright ? Solution: USD/INR spot = 48.75/48.90 2 month = 48.87/ 49.90. USD/JPY spot = 125.50/ 126.10 2 month = 125.30/ 125.95 INR & JPY = 125.30/48.87 = 2.56. 125.97/ 49 = 2.57. JPY = 2.56/2.57 for 1 INR
  • 25. PROBLEM Give the following date, calculate the arbitrage possibilities Spot rate : 42.0010=1$ Forward rate (6month)= 42.8020Rs/1$ Annualised interest rate on 6 month Rs:12% Annualised interest rate on 6 month dollar:8%
  • 26. SOLUTION  The rule is that if the interest rate differential is greater than the premium or discount ,place the money in the currency that has a higher rate of interest or vice versa  Negative interest rate differential = 12-8=4%  Forward premium = forward rate –spot rate ------------------------------- X 100 spot rate 42.8020-42.001 12 ---------------------- x100 x -------- = 3.8141% 42.0010 6
  • 27.  Negative differential > forward premium . Hence, there is a possibility of arbitrage inflow in India.  Arbitrage possibility for an investment of $1000 by taking a loan @ 8% in US. An arbitrageur would invest in India at spot rate of RS 42.0010 @12% for six months and cover the principal +interest in six month forward rate  Principal:$1000= 42,001 Rs  Interest on investment for six month = Rs 42001 x 12/100 x 6/12 = 2520.06 So total amount at the end of six month+ principal +interest = 2520.06+ 42001= 44,521.06
  • 28.  Converting the above in dollars at the forward rate = 44,521.06/42.8020= $1040.16  The arbitrageur will have to pay at the end of six month = $ 1000 + ($1000 x 8/100 x6/100) = $1000+ $40 =1040$  Hench the arbitrageur gain + ($1040.16-$1040) =$0.16 on borrowing $1000 for six month
  • 29. PROBLEM: INTEREST RATE PARITY An American firm purchases $4000 worth of perfume (ff 20,000) from a French francs. The following quotation and expectations exist for the FF.  Present spot rate $ 0.2000, US interest rate15%.  90 day forward rate 0.2200, French interest rate 10% SR 90days = 0.2400.  What is the premium or discount on the forward French francs ? What is the interest differential between US and French ? Is there an incentive for covered interest arbitrage?  If there is CIA (covered interest arbitrage), how can an arbitrageur take advantage of the situation? Assume the arbitrageur is willing to borrow $4000 or FF 20,000 and there are no transaction costs.  If transaction costs are $50 , would an opportunity still exit for CIA?
  • 30. SOLUTION  Forward premium on FF = FR-SR/SR X 100 .22-.20 ------------ X 100 X 4 = 40% .20 Interest rate different is 15-10=5% . Yes there is an incentive for CIA (outflow of trends from US ) as interest differential in favour of France is -5% in flour of US 5%.
  • 31.  The arbitrageur can take advantage of the situation in the following manner. Borrow $4000 for 90 days. So amount is t paid after 90 days = 4000 (1+15% X1/4) =$4.150 Convert $4000into FF at current SR ie $1=5Ff $4000 = FF 4000x5 =FF 20,000 $4000= FF 20,000 Invest 20,000 in France @10% p.a for 90 days Amount received at the end of 90 days = FF 20,500
  • 32.  Sell investment proceed forward at rate FF1= $.22 Amount received in US $ after 90 days by selling FF 20500= $ (20500 x .22) = $4510 Amount received =4510$ Amount paid =4150$ ------------ Profit = 360$  As the profit of 360$ > transaction cost of 50 $ opportunity still exist for a CIA.
  • 33. PROBLEM Dm spot was quoted 0.40$ in New York, the price of the pound sterling was quoted $1.80. 1. what would you expect the price of the pound to be in Germany? 2. If the pound were quoted in Frankfort at Dm 4.40/pound what would you do to profit from the situation?
  • 34. SOLUTION  1Dm =$ 0.40 ; 1 pound = $1.80 1/0.40Dm = 1/1.80pound 1 pound = 1.80/0.4 Dm = 4.5Dm = 4.5 Dm/pound.  Buy 1 pound for Dm 4.40 in Frankfort . With this pound sell in NY: for $1.80. with $1.80 , buy 4.5 Dm . Hench the arbitrager will profit 0.1 Dm for every 4.40 Dm
  • 35. PROBLEM  You have called your foreign exchange market and asked for quotations on the spot , 1 month, 3 months, 6 months. The trader has responded with the following. = $ 0.024.479/81 3/5 8/7 13/10 1. What does this mean in terms of dollars per euro 2. If you wished to buy spot Euros how much would you pay in dollars? 3. If you wanted to purchase spot US$ how much would you pay in euro? 4. What is the premium or discount in the 1, 3, 6 months forward rates in annual percentages? ( assume you are buying in Belgian francs) 5.
  • 36. SOLUTION Spot rate Bid price Ask price Spot rate $0.2479 $0.2481 1 month $0.2482 $0.2486 3 months $0.2471 $0.2474 6 months $0.2466 $0.2471 2. $ 0.2481 Will Ask Price 3. US $ spot = 1/ 0.2481 = 4.034
  • 37.  Premium or discount rates in forward market (assuming that buying in Belgian francs) 0.2486 – 0.2481 1 month = ------------------------ X 12 X100 = 2.42% forward 0.2481 premium 0.2474 – 0.2481 12 3 month = ------------------------ X 100 X ----- =1.13% Forward 0.2481 3 Discount 6 month = 0.2471- 0.2481 12 Forward --------------------- X 100 X ----- = 0.8% Discount 0.2481 6
  • 38. PROBLEM  Spot rate = 44.0030 = $1  6 month FR = 45.0010 = $1  Annualised interest rate on 6 month rupee = 12%  Annualised interest rate on 6 month dollar = 8% Given the above data is there arbitrage possibility?
  • 39. SOLUTION  6 month forward US $ is being quoted at a premium as follows 45.0010- 44.0030 12 ------------------------- X ------ X100 = 4.5361% 44.0030 6 Interest rate differential = 12-8 = 4% Since interest rate differential is smaller than the premium , it would be beneficial to place money in US $ as 6 month interest rate is lower .
  • 40. An arbitrageur would take the following steps: 1. Borrow Rs10,000 at 12% for 6 months 2. Convert this SR to obtain US$ 227.257 (10,000/44.0030) 3. Invest $ at 8% in money market for 6 months to receive 6 1 = $ 227.257 8 X ----- X ----- + 1 12 100
  • 41. 4. Sell Us$ at 6 month forward to receive = 236.3473 X 45.0010 = Rs 10635.865 5. Return the rupee debt borrowed at12 % . The amount to be refunded is Rs 10,600 6 1 =10,000 1+ 12 X ---- X ----- + 1 12 100 Profit = amount received – amount borrowed = 10,635.865 -10,600 = 35.865
  • 42. PROBLEM  From the following data calculate the possibilities of gain /loss in arbitrage. Spot rate FFr 6.00 = $1 , 6 month forward rate FFr 6.0020 = $1. Annualised interest rate on 6 month s US$ = 5% Annualised interest rate on 6 month Fr = 8% (It Direct Quote Method)
  • 43. SOLUTION  Negative interest rate differential = 5 - 8 = -3% forward Premia (annualized ) = FR-SR/SR *100 * 12/6 = (6.0020 -6.0000 /6.0000) x100 x 12/6 = 0.67% Here we find that the negative interest rate differential > forward Premia , Hench there will be arbitrage inflow in France
  • 44.  The arbitrage possibility will be shown below. 1. Assume an arbitrageur borrows $1000 for 6 months. Amount to be repaid at the end of 6 months will be = $1000 +$ 1000 x5/1000 x6/12 = $1025. 2. The arbitrage would then convert $ into FF at spot rate and invest the amount in France @8%. Converting $ into FF at the spot rate $100 –FF 6000. Invest in France @8% = 6000 x 8/100 x 6/12 = 240 Total amount received is FF 6000 + FF240 = FF 6240. convert Ff6240 into $ at the forward rate at the end of 6 month = FF 6240 /FF 6.0020 = $ 1039.65. Amount received by the arbitrageur = $1039.65 Amount paid = $ 1025.00 ----------------------- profit = $ 14.65
  • 45. Problem:  If the current Rs/$ rate is Rs 50, one year inflation rate is 10% for Rs and 2% for dollar. If 1 year later , the rate of exchange between $ & Rs in forward rate is 55, is there scope for arbitrage , if PPP is applicable?  Spot exchange rate between Rs/$ 50, if annual inflation rate is 10% in India and 3% in us what will be the rate of return to Indian investor if the return on us security is 10%if PPP is applicable?  In the above case if the percentage of return in India is 10%. What is the rate of return to American investor?
  • 46. Sol :1: S1A/B = S0A/B X (1+rA/1+rB) S1 = Future Exchange Rate rA = Interest rate of country A rB = Interest rate of country B As per PPP one year later rate will be = (50* 1.10)/1.02 = 53.92. Since the present forward rate is not in alignment he can sell the $ forward  Sol: 2: Indian investor can convertRs50 to one dollar and invest the same in Us security. After one year he will get 1.10 $ .  One year later the= $=(50* 1.10)/1.03 =53.40 Rs.  Sol:3: American investor can convert one dollar to 50Rs and invest the same in Indian market. After one year he will get Rs 50 * 1.10 = Rs55, if this is converted to dollar it will be 1.03$ . So the rate will be 3%for American investor.
  • 47. PROBLEM An Indian company ,AB ltd imports machinery worth ₤2.0 million and is to make the payment after 6 months. The current rate are Spot rate = Rs 66.96/₤ 6 month forward rate = Rs 67.50 /₤  What should AB ltd do if they expect that in six month time the pound will settle at Rs67.15/₤?  What are the option available to the company in case of an expected appreciation / depreciation in the rupees?
  • 48. SOLUTION  Spot rate ₤1 = Rs 66.96  6 month forward rate ₤1 = 67.50  Expected spot rate after 6 month ₤1 = Rs 67.15 1) Since AB ltd has a liability in foreign currency pound , they are importing a machinery worth ₤ 2.0 million . Both the market and the company expect the pound to appreciate. 2) Hench company should estimate the relative cost of hedging and if it is not high , the company should hedge its payments. 3) Incase of depreciation of pound the company need not do anything as it stands to gain . In case of appreciation of pound it should hedge its payments as the company will be exposed to exchange rate risk
  • 49. PROBLEM  The following data is given: Spot rate FFrl = 66.60 Rs 6 month forward rate FFrl = Rs 6.85 FFr interest = 8.3% Rs interest = 10.5 % Analyze the different arbitrage possibilities
  • 50. SOLUTION  Given the above data if one invest money in India , he gets = ( 1+ .105/2) =1.0525 after 6 months.  If one invest money in FFr , he gets 6.85/6.60 (1+0.83/2)= Rs10809  So there will be an arbitrage outflow from india.
  • 51. PROBLEM  Set out below is a table of cross rates. Deutsch mark Dollar French franc Pound sterling Frankfurt ? 2.2819 0.4712 4.0218 New York 0.4421 ? 0.2110 1.8000 Paris 2.0949 4.7393 ? 8.4301 London 4.0207 1.7775 8.4232 ? For Frankfurt , New York and Paris all quotes for London all quotes are indirect. If all above quotes were available at the same time and assuming no transaction costs how might a trader take advantage of the situation
  • 52. SOLUTION Given the arbitrage opportunities with no transaction cost.  Arbitrage between Dm & pound : buy pound in Frankfort . For every Dm, one will get ₤4.0218. These can be sold in London to get Dm for which one will have to pay ₤ 4.0207. These can be sold in London to get Dm for which one will have to pay ₤ 4.0207.Thus there is a potential gain of 0.0011 pounds for every Dm.  Arbitrage between $ & ₤ : buy ₤ in New York and sell in London.  Arbitrage between FFr and ₤ : buy Paris and sell in London.
  • 53.  Arbitrage between $ & ₤ :buy $ in Frankfort and sell in New York . For every Dm one will get $2.2819. these can be sold in Frankfort and sell in New York . For every Dm one will get $2.2812. these can be sold in New York to get Dm for which one will have to pay ₤2.2619. thus there is a potential gain of $0.0193 for every Dm traded.  Arbitrage between Ff and Dm : buy in Paris and sell in Frankfort. For every FF one will get Dm 2.0949. these can be sold in Frankfort to get Ff for which one will have to pay Ff 0.9817. thus there is a potential gain of Ff 0.0128 for every Dm traded. Similar arbitrage opportunities will exist across all the currency combinations.
  • 54. PROBLEM  A foreign exchange trader gives the following quotes for the Belgian franc Spot , 1 month, 3month , 6 month to us based treasurer. = $0.02368/70 4/5 8/7 14/12. 1. Calculate the outright quotes for 1, 3, &6 months forward?. 2. If the treasurer wished to buy Belgium franc 3 month s forward how much would the pay in dollars ? 3. If he wished to purchase US$ 1 month forward how much would he have to pay in Belgium franc? 4. Assuming that Belgian franc are being bought what is the premium/ discount for the 1, 3, 6 months forward rate in annual percentage terms? 5. What do the above quotation imply in respect of the term structure of interest rates in the USA & Belgian?
  • 55. SOLUTION Bid Ask Spot $ 0.02368 $0.02370 1 month $0.02372 $0.02375 3 month $ 0.02360 $0.02363 6 months $0.02354 $ 0.02358
  • 56. 2. To buy Belgian francs 3 month forward the treasurer has to pay $ 0.02363. 3.US $ 1 month forward = 1/0.02375= Belgian franc 42.10526. For 1 month forward premium = (0.02372-0.02368)/0.02368 x 100 x 12=2..27% p.a. For 3 month forward discount = (0.02360- 0.02368)/0.02368 X100 X12/3 = 1.35% p.a. For 6 month forward discount = (0.02354- 0.02368)/0.02368 x100 x 12/6 = 1.185 p.a. 4. Belgian interest rates are expected to rise and US interest rate are expected to fall.
  • 57. PROBLEM  “Compaque” company has to make a US $ 1 million payment in 3 months time . The dollars are available now . You decide to invest them for 3 months. US$ deposit rate :9% p.a. UK deposit rate :10% p.a. Present spot rate is $ 1.90/pound. 3 month forward rate is $ 1.88/₤.  Where should the company invest for better returns?  Assuming that the interest rates and the returns spot exchange rate remain as above what forward rate would yield an equilibrium situation?  If the sterling deposit rate was 12% p.a. and all other rates remain as in the original question, where should you invest?
  • 58. SOLUTION  Alternative :1 1. invest US$ 1 million in Us @9% p.a. for 3 months. Interest earned = ( 100,000 x.09 x 3/12) - $ 22,500.  Alternative :2 1. sell the US $ 1 mn and buy pound from the spot market , we get = 1,000,000/1.90 = ₤ 526,315.789. 1. Invest the available ₤ @10% for 3 months yield an interest = 526,315.789 x 10 /100 x 3/12 = 13,157.895. 2. principal + interest after 3 month = 526,315.789 + 13,157.895 = ₤ 539,473.684. 3. selling the pound to buy US $ we get 539,473.684 x 1.88 = $ 1,014,210.526 4. Hench income = $1,014, 210.526- $1,000,000 = $14210.526. Hench the company should invest in the US as it results in better return of US$ 8,290.
  • 59.  Assume the forward rate be ‘x’ For an equilibrium situation amount at the end of 3 month s should be equal . Hench amount invested in sterling covered by forward rate should be = $ 1,022,500. Hench x = 1022,500/539473.684 = 1.895 Hench forward rate = $ 1.89 / ₤.  Sell the US$ and buy ₤ from the spot market we get = 1,000,000/1.90 = ₤ 526,315.789 1. invest the available ₤ @12% for 3 months yield an interest = 526,315.789 x 12 /100 x 3/12 = 15,789.473 2. Principal = interest after 3month = 526,315.789 = 15,789.473 = ₤ 542,105.262. 3. Selling the pound to buy US$ we get = 542,105 .262 X1.88 = $ 1,019,157.893 Thus income (net) = 19,157.893. 1. since interest earned by investing in US$ = 22,500. Hench even if the sterling deposit rate become 12% p.a. investing US$ is still a more profitable alternative.
  • 60. PROBLEM 21  A MNC gives the following outright quotations for the Singaporean dollars: BID ASk Spot rates 1.2440 1.2450 1 month 1.2455 2.2475 3month 1.2477 1.2484 6 month 1.2482 1.2498
  • 61.  Calculate forward quotes for the Singaporean dollar as an annual percentage premium or discount if you reside in US.  calculate the annual percentage premium or discount on the US dollar for each forward rate for a foreign exchange trader residing in singapore?
  • 62. SOLUTION  We are buying USD. BID ASK Premium on buying USD annualized Spot rate 1.244 1.245 1 month 1.2455 1.2475 0.200803214 2.409638554 3 month 1.2477 1.2484 0.072144289 0.288577154 6 month 1.24 82 1.2498 0.112143544 0.224287087
  • 63. Bid Ask % discount on buying USD annualized Spot rate 0.80385852 0.803213 1 month 0.80289041 0.801603 -.0.002004008 -0.0240280 3 month 0.80147471 0.801025 - 0.000720923 -0.0028836 6 month 0.80115366 0.800128 -0.001120179 -.00022403
  • 64. PROBLEM  The direct quote in Tokyo for Peso is given as ¥28.8358/MP Bid & ¥ 28.8725/MP Ask, in México city 0.04418/0.04488.  Calculate the bid –ask Spread as percentage of bid price from the Japanese and from the Mexican perspective.  would there exist an opportunity for profitable arbitrage? If yes describe the necessary transaction assuming a ¥ 1 million starting amount.
  • 65. SOLUTION Direct terms Bid rate Ask rate In Japan ¥ 28.8358/MP ¥28.8725/MP In Mexico MP 0.04418/¥ MP 0.04422/¥  Bid-Ask- spread as a % in yen (Japanese) = (28.8725- 28.8358)/ 28.8358 = 0.127%.  Bid –Ask-spread as % in MP (Mexican) = (0.04422- 0.04418) / 0.04418 = -0.09%. Opportunities for arbitrage:( in Mexico & in Tokyo)  In Mexico= MP 0.04418/ ¥ Bid: MP 0.04422/ ¥. Or ¥22.6142/NP bid: ¥22.6346/MP ask
  • 66. Purchase MP in Mexico at ¥ 22.6346/MP (ask) Assuming ¥ 1 million as the starting amount & converting it to MP. so, MP received = 1m / 22.6346= 42625.7 MP.  In Tokyo, = ¥28.8358/MP bid and ¥28.8725/MP ask, convert MP into yen in Tokyo at ¥ 28.8358/MP bid.  Yen received = ¥1.229m  Profit = ¥ 1.229m- ¥ 1m = ¥.229m  Hence profit earned = ¥ 0.229m
  • 67. PROBLEM  call option on euro is written with a strike price of $ 0.9400/e at premium of 0.9000$ per euro/E and with an expiration data 3 month s from now . The option is for E 100,000. calculate your profit or loss if the excise before maturity at the time when the euro is traded spot at:  $ 0.9000/e  $ 0.9200/e  $ 0.9400/e  $ 0.9600/e  $ 0.9800/e  $ 1.0000/e  $ 1.0200/e
  • 68. SOL :CALL OPTION: FSP>SP =EXECUTE, PUT OPTION: FSP<SP = EXECUTE Spot rate Strike price Premium Decision Profit = spot rate – (strike price +Premium) $0.9000/e 0.9400/e 0.0090/e Execute option profit 0.049 $0.9200/e ,, ,, Execute option profit 0.029 $0.9400/e ,, ,, Execute option profit 0.009 $0.9600/e ,, ,, Not Execute 0.011 $0.9800/e ,, ,, Not Execute 0.031 $1.000/e ,, ,, Not Execute 0.051 $1.0200/e ,, ,, Not Execute 0.071
  • 69. PROBLEM  Spot and 180 day forward exchange rates of several major currencies are given . For each pair calculate the percentage premium or discount expressed as annual rate. Countries Quotation spot rate 180 days forward rate European euro $ 0.8000/e $0.8160/e British pound $ 1.562/₤ $1.5300/₤ Japanese yen ¥120.00/$ ¥118.00/$ Swiss franc SF 1.6000/$ SF 1.6200/$ Hong kong dollar HK $ 8.000/$ HK $7.8000/$
  • 70. SOLUTION  Forward premium/ discount = = FR-SR/SR X100 X360/number of days.  European Euro = 0.8160- 0.8000/0.8000 X 360/180X100 Premium = 4%  British Pound =1.5300-1.562/1.562 X 360/180 x 100 Discount = -4.08%  Japanese yen = 118.00-120.00/120.00 x360/180 x 100 Discount = -3.33%.  Swiss franc = 1.6200- 1.6000/1.6000 x 360/180 x 100 Premium = 2.5%  Honk kong dollar = 7.8000-8.000/8.000 x 360/180 x 100 Discount= -5%
  • 72. PROBLEM:1  Given the following date calculate the arbitrage possibilities Spot rate 1 $ = Rs 42.0010. 6 month forward rate 1$ = Rs 42.8020 Annualised interest rate on 6 month Rs:12 % Annualised interest rate on 6 month USD:8% Is arbitrage is possible.
  • 73. PROBLEM:2  Given the following data: Spot rate = 46.0010 Rs =1$ 6 month forward rate Rs 46.8020 =1$ Annualised interest rate on 6 month Rs = 12% Annualised interest rate on 6 month $ =8% Initial investment is 10,000$ Calculate the arbitrage possibilities.
  • 74. PROBLEM:3  USD/INR spot rate = 53.75/90. 2 month swap =.12/.20  USD/JPY spot rate = 110.50/125.45 . 2 month swap =.20/.15  Find INR/JPY , 2 month outright
  • 75. PROBLEM: 4  The current CHF/USD spot =0.6675. the following 90 day call option on CHF is available. Strike price Premium 0.60 0.075 0.65 0.03 0.68 0.01 0.70 0.005 0.75 0.002 Your view is that CHF is going to make strong up move during the next 90 days your risk appetite is moderate . What strategy is suitable for you/ explain with the pay off table
  • 76. EXOSURE : TRANSLATION , TRANSACTION, OPERTING
  • 77. PROBLEM:  Farm product is the Canadian affiliate of a US manufacturing company .its balance sheet in thousand of Canadian $ for dollars for January, the January 1 20x2, exchange rate was C $ 1.6/$. Farm product balance sheet (thousand of C$) Assets Liabilities and networth Cash =C$ 100000 Account receivable = 220000 Inventory = 320000 Net plant and equipment = 200000 --------------- Total 8,40,000 Current liabilities =C $ 60,000 Long term debt = 160,000 Capital stock =620000 ---------------- total 8,40,000
  • 78.  Determine farm product accounting exposure on January 1 2012, using the current rate method/ monetary /non monetary method.  Calculate farm product contribution to its parent accounting loss if the exchange rate on December 3 20x2 was C$ 1.8 per $. Assume all accounts remain as they were at the beginning of the year.
  • 79. SOLUTION Current /non current method C$ Exchange rate Conversion to US$ on jan 1 20x2 Assets 1. Account receivable 2. cash 3. Inventory 4. Plant & equipment Liabilities 1. Current liabilities 2. Long term debt 3. Capital stock CTA 100,000 220,000 320,000 200,000 60,000 160,000 620,000 ======== 840,000 1.8 1.8 1.8 1.8 1.8 1.8 1.6 (100000/1.8) = 55,555.55 = 122,222.22 = 1,77,777.77 = 1,11,111.11 -------------------- = 4,66,666.66 ------------------- = 33,333.33 = 88,888.88 = 3,87,500.00 = (43,055.55) -------------------- 4,66,666.66
  • 80. As per current rate method Accounting exposure on Jan 1 20x2 is as follows Exposed asset = US$ 4,66,666.66. Exposed liabilities = US $ 1,22, 222.22. Accounting exposure = 3,44,444.45 Accounting loss as show as in CTA (Cumulative Translation Adjustment) account is =US $ 43,055.55  Cumulative Translation Adjustment :- Cumulative Translation Adjustments are an integral part of the financial statements for firms with international market exposure.  An entry in the comprehensive income section of a translated balance sheet summarizing the gains/losses resulting from varying exchange rates over the years. A CTA entry is required under the Financial Accounting Standards Board (FASB) No.52 rule as a means of helping investors differentiate between actual operating gains/losses and those generated via translation.  (Opening Functional Balance x (Current month-end rate - Previous month-end rate)] + [Sum(Transactions in month) x (Current month-end rate - current average rate)] ) x [-1]
  • 81. Monetary/non monetary method C$ Exchange rate Cinversion to US $ on jan 1 20x2 • Cash • Account receivable • Inventory • Plant & equipment • =========== • Current liabilities • Long term debt • Capital stock CTA 100,000 220,000 320,000 200,000 ========= 60,000 160,000 620,000 1.8 1.8 1.6 1.6 1.8 1.8 1.6 55,555.55 122,222.22 200,000.00 125,000 ========== 5,02,777.77 33,333.33 88,888.88 38,7500. (6,944.44)
  • 82. AS PER THE MONETARY/ NON MONETARY METHOD  Accounting exposure on jan /1/20x2 Exposure assets= US $ 1,77,777.77 Exposure liabilities =US $ 1,22,222.21 Accounting exposure = 55555.57 Accounting lose as on CTA a/c =US $ 6,944.44
  • 83. PROBLEM ABC house ltd manufactures orange marmalade in England .it is the wholly owned subsidiary of XYZ Inc. of USA the financial currency for ABC is the pound sterling which currently sells at $1.5000/£. The reporting currency for XYZ is the US $ . Non- consolidated financial statement for both ABC and XYZ are as follows ( in thousand )
  • 84. ASSETS XYZ ABC LIBILITES XYZ ABC • CASH • ACCOUNT RECEIVABLE • INVENTORY • NET PLANT & EQUIPMENT • INVESTMENT $ 8000 10,000 8,000 10,000 4,500 ₤ 2000 4000 2000 6000 ---- • CURRENT LIABILTIES • 5 YEAR TERM LOAN • CAPITAL STOCK • RETAINED EARNING $ 22,000 ---- 9,000 9,500 £ 4,000 4,000 2,000 4,000 Prepare a consolidation balance sheet for XYZ ltd. What is ABC LTD accounting exposure in dollars ? Use the current rate method of calculation. Before n business activates take place the pound sterling deprecation 9% in value relative to the dollars what is the new spot rate? What is XYZ accounting loss or gain if any by the current rate method / Monterey non monetary method
  • 85. SOLUTION  Balance sheet for ABC ltd in dollar Assets ABC Rate $ Liabilities ABC Rate $ CASH ₤2000 1.5 3000 CURRENT LIABILTIES £ 4000 1.5 6000 ACCOUNT RECEIVABLE 4000 1.5 6000 5 YEAR TERM LOAN 4000 1.5 6000 INVENTORY 2000 1.5 3000 CAPITAL STOCK 2000 1.5 3000 NET PLANT & EQUIPMENT 6000 1.5 9000 RETAINED EARNING 4000 1.5 6000 14000 21000 14000 21000
  • 86. CONSOLIDATION BALANCE SHEET FOR XYZ AND ITS SUBSIDIARY ABC LTD Assets Amount Liabilties Amount CASH $11000 (8000+3000) Current liabilities $ 28000 (22000+6000) ACCOUNT RECEIVABLE 16000 (10000+6000) 5 year term loan 6000 (0+ 6000) INVENTORY 11000 (8000+3000) Capital stock 12000 (9000+3000) NET PLANT & EQUIPMENT 19000(10000+9 000) Retained earning 15500 (9500+6000) INVESTMENT 4500 Total 61500 Total 61500
  • 87. B. USING THE CURRENT RATE METHOD Assets Amount Cash $ 3000 Accounts receivable $ 6000 Inventory $ 3000 Net plant & equipment $ 9000 total $ 21000 Liabilities tterm loan Amount Current liabilities $ 6000 5 yrs term loan 6000 Capital stock 2000 Retained earning 4000 total 18000
  • 88. CONTI…………….  Accounting Exposure (21000-18000) = $3000.  New position of the firm after deprecation Assets Current rate method m/nm amount Cash $ 2730 (3000*9%)-3000 $ 2730 Account receivable $ 5460 (6000*9%)-6000 $ 5460 Inventory $ 2730 (3000*9%)-3000 $ 3000 Net plant equipment $ 8190 (9000*9%)-9000 $ 9000 Total $ 19110 $ 20190
  • 89. Liabilities & net worth Current liabilities $ 5460 $ 5460 5 yrs term loan $ 5460 $ 5460 Capital stock $ 3000 $ 3000 Retained earning $ 6000 $ 6000 CTA (810) 270 total $19110 $ 20190 Loss by current method =$ 810 Gain monetary/non monetary method =270$
  • 90. PROBLEM  New Haven a dealer based in Europe is owned by an MNC inc. of the united states. Given below is new haven’s balance sheet at the current exchange rate of $1.50/ euro. Value in euro At value at $ 1.5/euro Assets Cash & short term securities $ 50,000 $ 75,000 Account receivable $ 30,000 $ 45,000 Inventory $ 20,000 $ 30,000 Plant equipment $ 600,000 $ 900,000 Total assets $700,000 $10,50,000
  • 91. Liabilities Value in euro Value at $1.5/e Account payable $ 150,000 $ 225,000 Short term debt $ 60,000 $ 90,000 Long term debt $ 410,000 $ 615,000 Net worth $ 80,000 $ 120,000 Total liabilities and net $7,00,000 $ 10,50,000 worth For the current /non current method the temporal method and the all current rate method calculate . a. The company’s exposed assets exposed liabilities and net exposed assets under each accounting translation method b. Suppose the euro depreciate by25% identify the impact of 25% depreciation of the euro on new haven’s consolidation balance sheet under each accounting translation method.
  • 92. SOLUTION Value in euro Value at 1.50 (in $) Assets; • Cash and short term securities • Account receivable • Inventory • Plant and equipment Total assets Liabilities • Account receivable • Short term debt • Long term debt • Net worth Total liabilities & net worth 50,000 30,000 20,000 600,000 700,000 150000 60,000 410000 80,000 700,000 75000 45000 30000 900,000 1050000 2,25000 90,000 615,000 120,000 1050,000
  • 93. Deprecation = 25% initial euro = 1.5$ that is 1 $ = .67 euro, Deprecation of 25% implies 1 $ = 1.25 x .67 euro =.83 which is same as 1 euro = 1.2 $. All current method:  Expose assets: 8,40,000$  Expose liabilities : 744,000$  Net exposed assets: 96,000$ Current /non current method:  Expose assets : 120,000$  Expose liabilities: 252000$  Net exposed asset: =132,000$ Temporal method:  Exposed assets: 120,000$  Exposed liabilities: 744,000$  Net exposed assets: = -624,000$
  • 94. Balance sheet In euro Ex . rat e C /non C ($) Ex. rate Tempora l ($) Ex. rate Cur. Rate ($) Assets: • Cash & short term securities. • Account receivable • Inventory • Plant & equipment Total asset Liabilities • Account payable • Short term securities • Long term securities • Net worth Total liabilities Translation exposure 50,000 30,000 20,000 600,00 700,000 150,000 60,000 410,000 80,000 700,000 1.2 1.2 1.2 1.5 1.2 1.2 1.5 1.5 (50,000x1.2) 60,000 36000 24000 900,000 1020,000 180000 72000 615000 120000 987000 33000 1.2 1.2 1.2 1.5 1.2 1.2 1.2 1.5 60,000 36000 24000 900,000 1020,000 180,000 72,000 492,000 120,000 864000 156000 1.2 1.2 1.2 1.2 1.2 1.2 1.2 1.5 60,000 36000 24000 720,000 840,000 180000 72000 492000 120000 864000 -24000
  • 95. PROBLEM  An MNC has account receivable of $ 1.8 billion and account payable of $940 million . It also has borrowed $ 700 million . The current spot rate is $ 1.81138/£  What is the MNC dollar transaction exposure in dollar terms? In pound term?  Suppose the pound appreciates to $ 2.1122 /₤ what is the MNC gain or loss in pound terms , on its dollar transaction exposure?
  • 96. SOLUTION In dollar term: Functional currency = dollar (i.e. foreign currency so the method used would be current rate method) Net exposure = exposure asset – exposure liabilities. Current method Exposure assets = $ 1800 mn Exposure liabilities = $ 940+ $700= $ 1640 Translation exposure = 1800-1640 = $ 160 mn
  • 97.  In pound term Translation exposure = 1800- 1640 = $ 160 mn Current rate = 1.8138/£ so translation exposure = 160/1.8138 = ₤ 88.21mn b. The pound appreciation to $ 2.1122/£ Net exposure = ₤ 160 mn Current rate = $ 2.1122/£ so translation exposure = 160/2.1122 = ₤ 75.75 So this is a loss of (£ 88.21 – £ 75.75) = £12.46 mn
  • 98. PROBLEM A foreign company expect to receive Mexico $ 15 million entertainment fees from Mexico in 90 days . The current spot rate is $0.2320/M$ and the 90 day forward rate is 0.2240/M$.  What I the company’s peso transaction exposure associated with this fee?  What is the expected US $ value of the fee if the spot rate expected in 90 days in $0.2305. also calculate the hedged dollar value of the fee?
  • 99. SOLUTION  Mex $ received if the fee to be paid today = $ million 15 x .2320 = US $ 3.48 million  Mex $ received by the foreign company if the fee is to be paid in 90 days = 15 x .2240 = US $ 3.36 million  Transaction exposure = 3.36 - 3.48 = -0.12 million B. The expected US$ fee = 15 x .2305 = Us $ 34,57,500  The hedged dollar value of the fee is = 3480,000 – 3457000 = $ 23,000.
  • 100. PROBLEM  AV ltd is the Indian affiliate of affiliate of a US sports manufacture . AV ltd manufacture items which are sold primarily in the united states and Europe .AV balance sheet in thousand of rupees as of march 31 is as follows: Assets Liabilities Cash $ 6000 Account receivable $ 4500 Inventory $ 4500 Net plant equipment $ 10,000 Total $ 25000 Account payable $ 3500 Short term bank loan $ 1500 Long term loan $ 4000 Capital stock $ 10,000 Retained earning $ 6000
  • 101. Exchange rate for translating the balance sheet into US $ are:  RS 35/$ : historic exchange rate , at which plant and equipment long term loan and common stock were acquired or issued.  RS 40/$ : march 31 exchange rate this was also the rate at which inventory was acquired.  RS 42/$ April 1 exchange rate , after devaluation of 20%.  Assuming no change in balance sheet a/c between march 31 and April 1 calculate accounting gain /loss by the current rate method and by monetary / non monetary method explain accounting loss in term of change in the value of exposed accounts
  • 102. SOLUTION Exchange rate:  RS 35/$ historical rate for the others  RS 40/$ for inventory march 31 – historical rate for inventory.  RS 42/$ for April 1 – current rate as on April 1
  • 103. Rs Ex. Change rate Conversion to $ on Nov 31 Ex. Change rate Conversion to $ on April 1 • Cash • A/c Receivable • Inventory • Plant & equipment • Total • Account payable • short term loan • Long term loan • Capital stock • retained earning • CTA • Total 6000 4500 4500 10,000 25000 3500 1500 4000 10,000 6000 40 40 40 40 40 40 40 35 35 $ 150 $112.50 $112.50 $250.00 $625.00 $87.50 $37.50 $100.00 $285.71 $171.43 (57.14) $625.00 42 42 42 42 42 42 42 35 35 $ 142.50 $107.14 $107.14 $238.10 $595.24 $83.33 $35.71 $95.24 $285.71 $171.043 (76.19) $595.24
  • 104. ACCOUNTING LOSS BY CURRENT RATE METHOD IS $ 57.14 ON MARCH 31 March 31 April 1 Exposed assets $ 625.00 $ 595.24 Exposed liabilities $ 225.00 $ 214.28 Net exposed $ 400.00 $ 380.96
  • 105. Monetary /non monetary method Rs Ex. Change rate Conversion to $ on Nov 31 Ex. Change rate Conversion to $ on April 1 • Cash • A/c Receivable • Inventory • Plant & equipment • Total • Account payable • short term loan • Long term loan • Capital stock • retained earning • CTA • Total 6000 4500 4500 10,000 25000 3500 1500 4000 10,000 6000 40 40 35 35 40 40 40 35 35 $ 150 $112.50 $128.50 $285.71 $676.78 $87.50 $37.50 $100.00 $285.71 $171.43 (5.36) $676.78 42 42 35 35 42 42 42 35 35 $ 142.50 $107.14 $128.57 $285.71 $664.29 $83.33 $35.71 $95.24 $285.71 $171.043 (7.13) $664.29
  • 106. MONETARY / NON MONETARY METHOD  Accounting loss by monetary / non monetary Is = $ 5.36 on march 31,= $ 7.13 on April 1 March 31 April 1 Exposed assets $ 262.50 $ 250.00 Exposed liabilities $ 225.00 $ 214.28 Net exposed $ (37.5) $ (35.72)